#8 Rosh Review

Ace your homework & exams now with Quizwiz!

Question: Which potassium-sparing diuretic is a common risk factor for the development of urolithiasis?

Answer: Triamterene (Dyrenium®, Maxzide®, Dyazide®). Rapid Review Nephrolithiasis Patient will be complaining of flank pain radiating to groin PE will show a patient that won't lay still and hematuria Diagnosis is made by helical CT Most commonly caused by calcium oxalate Struvite: staghorn calculi, urease producing bacteria Uric acid: radiolUcent on xray, gout Cystine: children with metabolic diseases Most common location is the ureterovesiclular junction (UVJ) Treatment is: < 5 mm: likely to pass spontaneously > 8 mm: unlikely to pass, lithotripsy

Question: True or false: All patients aged 40 years and older with diabetes mellitus should receive statin therapy?

Answer: True.

Question: What type of Salter-Harris fracture is commonly missed on X-ray?

Answer: Type I.

Question: Do individuals with schizophrenia have an increased risk of violence?

Answer: Yes, especially those with uncontrolled paranoia and disorganized symptoms. Rapid Review Schizophrenia ≥ 2 symptoms: Delusions, hallucinations, disorganized speech, grossly disorganized or catatonic behavior, negative symptoms Brief psychotic disorder: < 1 month Schizophreniform disorder: 1-6 months Schizophrenia: > 6 months Schizoaffective disorder: psychosis + mania or depression

Question: Does bacterial vaginosis increases a woman's susceptibility to other sexually transmitted infections, such as herpes simplex virus and HIV?

Answer: Yes. Rapid Review Bacterial Vaginosis Patient will be complaining of malodorous vaginal discharge PE will show thin, gray/white discharge Labs will show pH > 4.5, clue cells Diagnosis is made by KOH to smear → fishy odor, "whiff test", Amsel Criteria Most commonly caused by Gardnerella vaginalis Treatment is metronidazole

Question: Is home monitoring recommended for patients with a history of severe BRUE?

Answer: Yes. Unlike in SIDS, where monitoring plays no role, home monitoring is recommended for use in patients with severe BRUE. Rapid Review Brief resolved unexplained events (BRUE) [formerly ALTE] Frightening event for observer + ≥ 1 of the following: ​Apnea Color change Muscle tone change Choking/gagging ​1-4 months Causes: idiopathic > GERD, seizure, LRI Admit most

Question: What is the normal opening pressure in adults?

Answer: 5-20 cm H20 Rapid Review Bacterial Meningitis (Adult) Streptococcus pneumoniae (most common) Headache, neck stiffness, photophobia, phonophobia, fever Meningismus, jolt accentuation, Brudzinski's sign (flex neck), Kernig's sign (extend knees) Acute: (< 24 hrs) bacterial Subacute: (1-7 days) lymphocytic/viral Chronic: (>1 week) insidious fungal (Cryptococcus in HIV patients) Empiric ABX before CT/LP Steroids Prophylaxis (rifampin) of close contacts (N. meningitidis and H. influenzae) Empiric antibiotic by age: 18 - 50 years: ceftriaxone + vancomycin > 50 years: ceftriaxone + ampicillin + vancomycin

Question: What is the CURB-65 score?

Answer: It attempts to predict the severity of pneumonia to help determine inpatient vs. outpatient treatment. Rapid Review Legionella Pneumonia Patient will be complaining of fevers, malaise, myalgias, cough and GI symptoms Labs will show leukocytosis, elevated liver transaminases and hyponatremia CXR will show unilateral patchy alveolar lower lobe infiltrates Most commonly caused by gram-negative bacillus and is found in aquatic environments Treatment is azithromycin

Question: Are individuals with bulimia typically underweight or overweight?

Answer: Most bulemics maintain normal or near-normal body weight. Rapid Review Anorexia Nervosa Patient will be an adolescent female With a history of excessive dieting, purging, body image disturbance, and fear of weight gain Complaining of amenorrhea PE will show > 15% below ideal body weight, lanugo, osteoporosis Treatment is behavioral therapy Comments: Patient will be in denial of illness

Question: What is the etiology of systemic lupus erythematous?

Answer: Multifactorial including genetics, environmental factors, hormones, race, medications and immunology. Rapid Review Systemic Lupus Erythematosus (SLE) African-Americans, females Malar rash Arthritis Renal disease Cardiac: fibrinous pericarditis, Libman-Sacks endocarditis CNS: HA, stroke, seizures Drug induced: Hydralazine, INH, Procainamide, Phenytoin, Sulfonamides (HIPPS) Antinuclear antibodies (ANA): 100% sensitive, not specific Anti-dsDNA antibodies: 100% specific, poor prognosis Anti-Smith antibodies: 100% specific, not prognostic Antihistone antibodies: sensitive for drug-induced lupus NSAIDs, steroids, immunosuppressants, hydroxychloroquine

Question: What is the most common cause of viral sialoadenitis?

Answer: Mumps parotitis, which, in contrast to bacterial parotitis, is usually bilateral. Rapid Review Sialolithiasis Submandibular gland (Wharton duct) Swelling, pain, worse with meals Warm compresses, gland massage, sialogogues

Question: What are the 3 values used to diagnose diabetes mellitus type 2?

Answer: Need 2: Fasting blood sugar > 125 mg/dL, HBA1c of 6.5%, glucose tolerance test results > 200 mg/dL or a random plasma glucose level > 200mg/dL + symptoms of hyperglycemia.

Question: What age groups are at higher risk for a malignant cause of their back pain?

Answer: Patients under 18 years or over 50 years of age. Rapid Review Back Pain Night pain, weight loss: malignancy Back pain + fever + neurological deficits: epidural abscess Acute bony tenderness: fracture Young, morning stiffness: seronegative spondyloarthropathy Urinary retention: cauda equina syndrome Pain with extension, relief with flexion: spinal stenosis Image if red flags present

Question: What qualifies as a complicated pneumonia?

Answer: Pneumonia involving a parapneumonic effusion or empyema, necrotizing pneumonia, lung abscess, or pneumatocele. Rapid Review Community-acquired Pneumonia (CAP) Patient will be complaining of sudden onset of cough, fatigue, and fever PE will show rales with auscultation of lung fields Most commonly caused by Streptococcus pneumoniae Treatment is amoxicillin

Question: What is the most common physical finding in Osgood-Schlatter?

Answer: Point tenderness on palpation of the tibial tuberosity. Rapid Review Osgood-Schlatter Disease Inflammation over insertion point of patellar tendon on tibial tuberosity Boys 10-15, athletes Pain/tenderness at tibial tuberosity Rx: rest, NSAIDs Self-resolving

Question: What diagnostic test, although low sensitivity, is often used when influenza is suspected?

Answer: Rapid influenza test.

Question: What are two clinical situations that can lead to a false-negative D-dimer?

Answer: Recent anticoagulation and subacute thrombosis (>7 days). Rapid Review Pulmonary Embolism 95% arise from deep leg veins Sudden onset of symptoms in 50% SOB, CP, tachypnea ECG: sinus tachycardia, nonspecific ST-T changes, right heart strain, S1Q3T3 (classic finding) CXR: nonspecific abnormalities, Hampton's hump (pleural-based wedge infarct), Westermark's sign (vascular cut-off sign) V/Q scan: usually nondiagnostic Low clinical suspicion: negative D-dimer excludes PE Dx of choice: CTPA Rx: anticoagulation, thrombolytics (if massive and HD unstable or submassive with shock, respiratory failure or evidence of moderate to severe RV strain), embolectomy (last resort)

Question: What therapy should household contacts of patients with confirmed meningitis receive?

Answer: Rifampin (oral). Ciprofloxacin (oral) and ceftriaxone are alternatives. Rapid Review Bacterial Meningitis (Adult) Streptococcus pneumoniae (most common) Headache, neck stiffness, photophobia, phonophobia, fever Meningismus, jolt accentuation, Brudzinski's sign (flex neck), Kernig's sign (extend knees) Acute: (< 24 hrs) bacterial Subacute: (1-7 days) lymphocytic/viral Chronic: (> 1 week) insidious fungal (Cryptococcus in HIV patients) Empiric ABX before CT/LP Steroids Prophylaxis (rifampin) of close contacts (N. meningitidis and H. influenzae) Empiric antibiotic by age: 18 - 50 years: ceftriaxone + vancomycin > 50 years: ceftriaxone + ampicillin + vancomycin

Question: What is the treatment of choice for Chlamydial urethritis?

Answer: Single dose azithromycin.

Question: How is scabies transmitted?

Answer: Skin-to-skin contact. Rapid Review Scabies Patient will be complaining of severe pruritus that is worse at night PE will show small papules, vesicles and burrows in the webbed spaces of the fingers and toes Diagnosis is made by microscopic visualization Most commonly caused by Sarcoptes scabiei hominis Treatment is permethrin 5%

Question: What is the gold standard test to diagnose celiac disease?

Answer: Small intestinal biopsy. Rapid Review Celiac Disease Patient will be complaining of diarrhea, steatorrhea, flatulence, weight loss, weakness and abdominal distension Labs will show IgA anti-endomysial (AGA) and anti-tissue transglutaminase (anti-tTG) antibodies Diagnosis is made by small bowel biopsy Treatment is gluten free diet Comments: associated with dermatitis herpetiformis (chronic, very itchy skin rash made up of bumps and blisters)

Question: What is the treatment for cases of mastoiditis that are refractory to parenteral antibiotics?

Answer: Surgical irrigation and debridement with possible mastoidectomy. Rapid Review Mastoiditis Untreated OM Otalgia, fever Mastoid erythema, tenderness Osteitis CN VII damage CT scan temporal bone IV ABX

Question: What is the psoas sign?

Answer: The psoas sign is pain that occurs in the right lower quadrant with passive right hip extension. Rapid Review Appendicitis Patient will be complaining of fever, pain that began periumbilical then moved to RLQ, nausea and anorexia PE will show Psoas sign (RLQ pain on extension of right hip), Obturator sign (RLQ pain on internal rotation of flexed right hip), Rovsing sign (right lower quadrant pain when the left lower quadrant is palpated) Diagnosis is made by ultrasound, CT Most commonly caused by fecalith Treatment is surgery

Question: What physical exam technique helps to differentiate ganglion cysts from other nodular lesions of the hand and wrist?

Answer: Transillumination. Rapid Review Ganglion Cyst Patient will be complaining of a painful mass in wrist Most commonly caused by repetitive activity causing tear or degeneration in joint capsule or tendon synovial sheath Treatment is observation or needle aspiration Comments: most common soft tissue tumors of the hand

Question: Which arbovirus is treatable with existing antiviral medications?

Answer: None. Rapid Review Encephalitis Arboviruses, HSV Fever, HA, AMS Psychiatric sx MRI

Question: What is the treatment for ethylene glycol poisoning?

Answer: Fomepizole.

Question: Why are children at increased risk for heat illness compared to adults?

Answer: Greater surface area to body mass ratio and produce greater heat/kg body weight, lower sweat rate, take longer to acclimatize to warmer environments, blunted thirst response.

Question: What is Munchausen syndrome?

Answer: A syndrome in which the patient deceptively exaggerates or feigns illness. This is an extreme form of factitious disorder.

Question: What are the clinical recommendations for preserving sperm count in patients with testicular carcinoma?

Answer: Because 45% to 55% of testicular cancer patients have azoospermia or oligospermia at or beyond 2 years after therapy, those patients who wish to preserve fertility should be offered semen cryopreservation before the start of therapy. Rapid Review Testicular Cancer Patient will be a man 20-35 years old Complaining of a testicular lump PE will show painless, hard, fixed mass Labs will show increased beta-hCG, alpha-fetoprotein (AFP), or lactate dehydrogenase (LDH) Diagnosis starts with ultrasound Comments: Most common risk factor: cryptorchidism Most common tumor: seminoma

Question: How is the Philadelphia chromosome identified?

Answer: Cytogenics or PCR for the bcr-abl gene product. Rapid Review Chronic Myelogenous Leukemia (CML) Patient will be a 30 - 60 years old Complaining of most patients asymptomatic when diagnosed PE will show splenomegaly Labs will show philadelphia chromosome t(9;22) (bcr/abl) and low leukocyte alkaline phosphatase (LAP) Treatment is allogenic HSCT (curative), imatinib Comments: Phase determined by blast percentage

Question: What type of venom does a brown recluse spider have?

Answer: Cytotoxic.

Question: Why are patients with Crohn's disease at risk for nephrolithiasis?

Answer: Fat malabsorption is common in Crohn's disease, leading to fat accumulation in the gut. Fat binds calcium leaving oxalate free to be absorbed and deposited in the kidney, where it can lead to stone formation. Rapid Review Nephrolithiasis Patient will be complaining of flank pain radiating to groin PE will show a patient that won't lay still and hematuria Diagnosis is made by helical CT Most commonly caused by calcium oxalate Struvite: staghorn calculi, urease producing bacteria Uric acid: radiolUcent on xray, gout Cystine: children with metabolic diseases Most common location is the ureterovesiclular junction (UVJ) Treatment is: < 5 mm: likely to pass spontaneously > 8 mm: unlikely to pass, lithotripsy

Question: Which of the hepatitis viruses is the only one which is a DNA virus?

Answer: Hepatitis B. Rapid Review Acute Hepatitis HAV: fecal-oral, shellfish, alone (no carrier), asymptomatic, acute HBV: HBsAg: active infection Anti-HBs: recovered or immunized Anti-HBc IgM: early marker of infection, positive in window period Anti-HBc IgG: best marker for prior HBV HBeAg: high infectivity Anti-HBeAb: low infectivity HCV: IVDA, chronic, cirrhosis, carcinoma, carrier HDV: dependent on HBV coinfection HEV: fecal-oral (enteric) high mortality rate among pregnant (expectant) patients, epidemics, HAV and HEV are fecal-oral: "The vowels hit your bowels" Autoimmune hepatitis: young females Alcoholic hepatitis: moderate transaminase elevation, AST>ALT Supportive rx

Question: Avascular necrosis is a complication of which traumatic hip injuries?

Answer: Hip dislocation or femoral neck fracture. Rapid Review Avascular Necrosis Causes: corticosteroids (most common), alcohol Most common site: femoral head MRI

Question: What are the Centor criteria for streptococcal pharyngitis?

Answer: History of fever, tonsillar exudates, tender anterior cervical adenopathy, absence of cough. Rapid Review Pharyngitis Viral > bacterial Bacterial GAS (S. Pyogenes) Centor criteria: fever, cervical lymphadenopathy, tonsillar exudate, no cough Rx: PCN, erythromycin (PCN allergic), steroids Complications: rheumatic fever, PSGN, abscess

Question: What is a common side effect of ceftriaxone in neonates?

Answer: Hyperbilirubinemia.

Question: What electrolyte must be given with potassium to ensure proper repletion?

Answer: Magnesium. Rapid Review Ileus Postoperative, electrolyte abnormalities Nausea/vomiting, constant pain Distension + ↓ bowel sounds

Question: Which common systemic diseases are associated with pericarditis?

Answer: Rheumatoid arthritis and connective tissue diseases. Rapid Review Pericarditis Patient will be complaining of pleuritic chest pain radiating to the back that is worse when lying back and improved when leaning forward PE will show tachycardia and pericardial friction rub ECG will show PR depression, PR elevation (aVR), diffuse ST segment elevation (concave) Most commonly caused by Idiopathic then viral (Coxsackie) Treatment is NSAIDs

Question: Where should subcutaneous injections be administered in infants younger than 12 months?

Answer: The anterolateral aspect of the thigh.

Question: What is the name of the ratio used in the work-up of ascites to help determine its cause?

Answer: The serum-ascites-albumin gradient or SAAG.

Question: What is the treatment of choice for chronic otitis media?

Answer: Tympanostomy tubes. Rapid Review Acute Otitis Media Patient will be an infant or young child Complaining of ear pain, fever, URI symptoms PE will show TM erythema and decreased mobility of TM Most commonly caused by viral > bacterial (S. pneumoniae most common) Treatment is amoxicillin

Question: What is the treatment for acute interstitial nephritis?

Answer: Withdrawal of offending medication.

Which one of the following is the most likely cause of acute kidney injury in a patient with eosinophiluria? Acute interstitial nephritis Ethylene glycol poisoning Poststreptococcal glomerulonephritis Rhabdomyolysis

Correct Answer ( A ) Explanation: Acute interstitial nephritis is an important cause of acute renal failure resulting from immune-mediated tubulointerstitial injury. The presence of eosinophiluria in a patient with acute kidney injury suggests acute interstitial nephritis, which is typically an allergic reaction to medications such as penicillins, sulfa-containing antibiotics and diuretics, NSAIDs and proton pump inhibitors. Patients with acute interstitial nephritis may also present with a rash, fever, eosinophilia, and other constitutional symptoms. The combination of elevated levels of creatine kinase or myoglobin, a dipstick positive for blood but negative for RBCs, and a history of muscle trauma would suggest rhabdomyolysis (D). Poisoning with ethylene glycol (B) or methanol should be suspected in a patient with acute kidney injury and altered mental status with an increased anion gap and osmolar gap. Ethylene glycol poisoning is associated with calcium oxalate crystals in the urine. An elevated antistreptolysin O titer when combined with the presentation of a full-blown nephritic syndrome with oliguric acute renal failure suggests poststreptococcal glomerulonephritis (C).

A 15-month-old old previously healthy boy presents to the office with 2-day history of right ear tugging and fever to 102°F. On otoscopy, his right tympanic membrane is erythematous and bulging. What is the first line treatment? Amoxicillin Amoxicillin-Clavulanate Oral decongestants Withhold antibiotics for 3 days to see if symptoms persist

Correct Answer ( A ) Explanation: This child has acute otitis media. The most common organisms that cause acute otitis media are non-typeable Haemophilus influenzae, Streptococcus pneumoniae, and Moraxella catarrhalis. The incidence of non-typeable Haemophilus influenzae is increasing while Streptococcus pneumoniae is decreasing. The first-line treatment in infants and toddlers (< 2-years-old) is amoxicillin because it covers both penicillin-susceptible and penicillin-nonsusceptible strains of S. pneumoniae as well as non-typeable H. influenzae that do not contain beta-lactamases. In patients with uncomplicated acute otitis media, treatment is warranted in individuals < 24 months of age, those with rapid onset of symptoms, and severe pain. Amoxicillin-Clavulanate (B) is reserved for children with acute otitis media who are not responding to first line treatment. Amoxicillin-Clavulanate covers beta-lactamase producing H. infleunzae as well as M. catarrhalis. Oral decongestants (C) have no proven value in in the treatment of acute otitis media. In addition, oral decongestants should not be used in children younger than 6 years of age and may even be harmful in infants younger than 6 months of age due to rebound nasal congestion that may impede respiratory function in infants who are obligate nasal breathers. Withholding antibiotic treatment for up to 3 days (D) to see if symptoms persist is accepted in uncomplicated patients over 24 months old. In these cases, you can provide a prescription for antibiotics and advise parents to wait up to 3 days. If symptoms persist, the parents can initiate treatment. The majority of cases of acute otitis media will resolve in two weeks without treatment.

A 35-year-old man presents to the Emergency Department four hours after being bitten on his left hand by a spider while cleaning out his shed. He currently complains of whole arm pain as well as headache, severe back spasms, and abdominal pain. What is the treatment of choice to control his symptoms? Intravenous calcium gluconate Intravenous diazepam Intravenous ketorolac Intravenous normal saline

Correct Answer ( B ) Explanation: Latrodectus, or "widow", spiders are most often found in woodpiles, basements, garages, and sheds. Latrodectus spiders aggressively defend their web, particularly when guarding eggs. Black widow bites occur most often between April and October and usually occur on the hands and forearms. Envenomation of a neurotoxic venom occurs when bitten, the most active component of which is α-latrotoxin. The neurotoxin causes receptor stimulation, pore formation, and release of neurotransmitters; predominantly acetylcholine and norepinephrine. Most Latrodectus bites are felt immediately as a pinprick sensation at the bite site followed by increasing local pain that may spread to include the involved extremity. A target lesion with a blanched center and surrounding erythema may develop at the site of the bite and frequently localized diaphoresis near the site of where envenomation occurs. The clinical syndrome with envenomation is referred to as latrodectism. Patients frequently complain of muscle cramps and spasms in large muscle groups including trunk, back, and abdomen, however, physical examination rarely reveals rigidity. If untreated, the pain with envenomation frequently resolves after one day but can persist for several days. Pain and muscle spasms can be effectively controlled with liberal doses of opioids and benzodiazepines. For severe envenomations, admission may be required for continued analgesia. The most effective therapies for severe envenomation are parenteral opioids and Latrodectus antivenin. Administration of Latrodectus antivenin often causes rapid resolution of symptoms and can significantly shorten the course of illness. However, this is reserved for the most severe cases. It was previously thought that intravenous calcium gluconate (A) was an effective treatment due to the calcium dependent pathway of α-latrotoxin. However recent evidence suggests that calcium is an ineffective treatment. Intravenous normal saline (D) may be part of an overall treatment strategy, but saline does not relieve the symptoms of a black widow spider bite. Intravenous ketorolac (C) is an analgesic which may provide some pain relief but does not adequately provide relief caused by severe muscle cramps and spasms associated with black widow bite envenomations.

A 47-year-old man presents to the emergency department due to acute, sharp chest pain that is relieved by leaning forward. A pericardial friction rub is auscultated on cardiac examination and an ECG shows diffuse ST elevations. Echocardiography reveals a small pericardial effusion. Which of the following is the most appropriate initial treatment? Atenolol Ibuprofen Nitroglycerin Prednisone

Correct Answer ( B ) Explanation: Pericarditis is defined as an inflammation of the pericardium and can be caused from an acute myocardial infarction, viral etiologies, medications, systemic disease and trauma to the chest cavity. A pericardial friction rub is often heard on auscultation and the patient presents with retrosternal chest pain that is relieved by leaning forward. Classic electrocardiographic changes include widespread concave upward ST-segment elevation without reciprocal T-wave inversions or Q waves. First-line treatment includes nonsteroidal anti-inflammatory drugs and colchicine. Beta-blockers such as atenolol (A) are used for cardiac arrhythmias such as atrial fibrillation. They are also used in the post myocardial infarction regimen as they help in the remodeling of the cardiac muscle. Nitroglycerin (C) is used for acute chest pain in the setting of a myocardial infarction as it reduces the afterload. Corticosteroids such as prednisone (D) are considered a second-line treatment for pericarditis when NSAID's and colchicine fail to relieve symptoms in severe or refractory cases that are often marked by systemic disease.

A 48-year-old woman presents with nausea and vomiting. She underwent an appendectomy 3 days ago. A contrast-enhanced CT of the abdomen and pelvis reveals an ileus. What electrolyte abnormality commonly causes an ileus? Hyperkalemia Hypernatremia Hypokalemia Hyponatremia

Correct Answer ( C ) Explanation: This patient is suffering from a postoperative ileus. The exact pathogenesis of postoperative ileus is unclear, but it appears to be associated with inhibitory neural reflexes, inflammation, and neurohumoral peptides. Hypokalemia is common in the postoperative period and may cause an ileus. Hypernatremia (B) and hyponatremia (D) do not cause changes to gut motility. Hyperkalemia (A) is associated with cardiac dysrhythmias but not ileus.

Which of the following types of leukemia is most associated with the Philadelphia chromosome? ALL AML CLL CML

Correct Answer ( D ) Explanation: Chronic myelogenous leukemia (CML), also called chronic myeloid leukemia, chronic myelocytic leukemia, and chronic granulocytic leukemia, is a clonal myeloproliferative disorder of the primitive hematopoietic stem cell that is characterized by overproduction of cells of the myeloid series, which results in marked splenomegaly and leukocytosis. Basophilia and thrombocytosis are common. A characteristic cytogenetic abnormality, the Philadelphia chromosome t(9;22), is present in the bone marrow cells in more than 90% of cases. Most patients (85 to 90%) present in the chronic phase. Eventually, if poorly controlled, CML evolves into the accelerated and blastic phases. CML constitutes one fifth of all cases of leukemia in the United States. It is diagnosed in 1 or 2 persons per 100,000 per year and has a slight male preponderance. This incidence of 5,000 to 6,000 cases annually has not changed significantly in the past few decades. The incidence of CML increases with age; the median age at diagnosis is 50 to 55 years. About 40 to 50% of patients diagnosed with CML are asymptomatic until the disease is found on routine physical examinations or blood tests. Peripheral smear shows increased WBCs and leukocyte alkaline phosphatase is low in contrast to other myeloproliferative disorders. The degree of leukocytosis correlates with tumor burden, as defined by spleen size. The symptoms of CML, when present, are due to anemia and splenomegaly; they include fatigue, weight loss, malaise, easy satiety, and left upper quadrant fullness or pain. Rarely, bleeding (associated with a low platelet count or platelet dysfunction) or thrombosis (associated with thrombocytosis or marked leukocytosis) occurs. Treatments include allogenic hematopoietic stem cell transplantation, imatinib mesylate (Gleevec®), and interferon-alpha. Acute leukemia is the result of a malignant event or events occurring in an early hematopoietic precursor. Instead of proliferating and differentiating normally, the affected cell gives rise to progeny that fail to differentiate but continue to proliferate in an uncontrolled fashion. As a result, immature myeloid cells in acute myeloid leukemia (AML) (B) or lymphoid cells in acute lymphoblastic leukemia (ALL) (A)—often called blasts—rapidly accumulate and progressively replace the bone marrow, diminishing the production of normal red cells, white cells, and platelets. CLL (C) is the most common leukemia (one third of all cases) in the Western world and is twice as common as CML. The disease occurs rarely in those younger than 30 years; most patients with CLL are older than 60 years. CLL increases in incidence exponentially with time; by age 80, the incidence rate is 20 cases per 100,000 persons per year. The male-female ratio is approximately 2:1.

A 27-year-old man presents with a testicular mass of unknown duration. The patient states he first noticed something unusual with his right testicle two weeks ago, but states he did not think it was urgent because it was not painful and believed it would resolve on its own. It has not changed since he first noticed the mass, and the patient still denies pain. On exam, the patient's right testicle is non-tender and a firm mass is felt. There is a negative transillumination test, and the mass is non-reducible. Which of the following is the best next step in management? Beta-hCG, alfa-fetoprotein, and lactate dehydrogenase levels Computed tomography of the abdomen and pelvis Needle biopsy Testicular ultrasound

Correct Answer ( D ) Explanation: The next step in evaluation of scrotal masses after the physical exam and transillumination test is testicular ultrasonography to help determine if the mass is extra-testicular or intra-testicular. Doppler should be utilized for the evaluation of both arterial and venous flow within the testicle. The differential for scrotal masses includes torsion, orchitis, epididymitis, hematocele, hydrocele, varicocele, hernia, and testicular cancer. Testicular cancer is the most common malignancy in males of age 15-35, the vast majority of which are germ cell tumors. It typically presents as a non-acute painless testicular mass, although some patients do report discomfort. Other manifestations may include gynecomastia, paraneoplastic hyperthyroidism, and paraneoplastic limbic encephalitis. Once diagnosed, the first step is radical inguinal orchiectomy with possible retroperitoneal lymph node dissection to evaluate for metastasis. Following surgery and staging, chemotherapy, radiation therapy, or active surveillance are all viable options. Needle biopsy (C) is not indicated in the workup of suspected testicular cancer. Imaging of the abdomen and pelvis (B) may be necessary once the diagnosis is established to evaluate for metastasis. Beta-hCG, alpha-fetoprotein (AFP), and lactate dehydrogenase (LDH) (A) are tumor markers seen in patients with testicular cancer and should be ordered if the ultrasound reveals an intra-testicular mass.

A 22-year-old man presents with a sore throat. On examination, the pharynx is erythematous without tonsillar enlargement or exudate. There is no cervical adenopathy. The patient is not sexually active. Which of the following is likely to provide the most improvement? Acetaminophen PO Amoxicillin PO Ceftriaxone IM Ibuprofen PO

Correct Answer ( D ) Explanation: The patient likely has viral pharyngitis. The examination demonstrates an erythematous pharynx without exudates and without adenopathy. In this case, the patient needs supportive care. NSAIDs, such as ibuprofen, will provide the most symptomatic relief. Acetaminophen (A) may provide some analgesic effect in this patient, but does not have the same effect as NSAIDs in improving resolution of symptoms as it lacks anti-inflammatory properties. Amoxicillin PO (B) may be helpful in the treatment of streptococcal pharyngitis although evidence does not suggest it improves time to resolution of symptoms or pain. In children, the use of antibiotics is recommended to prevent the possibility of later rheumatic heart disease. Ceftriaxone IM (C) is indicated for possible gonococcal pharyngitis, which does not have an exudative component. Clinicians cannot make the diagnosis without a careful sexual history.

A 57-year-old woman with cirrhosis complains of worsening distension of the abdomen and edema up to her lower legs despite compliance with eating less than 2 grams daily of sodium. She denies abdominal pain, constipation, dyspnea or fevers. On exam, her abdomen is distended and has a notable fluid wave. She has 2+ pitting edema to the level of her ankles. Which medication is the most appropriate next step in the management of this patient? Hydrochlorothiazide Lactulose Omeprazole Spironolactone

Correct Answer ( D ) Explanation: This patient has worsened ascites secondary to portal hypertension as a result of cirrhosis. Portal hypertension leads to ascites through a series of hemodynamic changes resulting in the movement of fluid into the peritoneum. The renin-angiotension-aldosterone system is activated leading to sodium retention and additional fluid accumulation. Spironolactone is an aldosterone antagonist, helping to reverse sodium retention, conserve potassium and achieve diuresis. The starting dose is 100 mg daily, with titration up to 400 mg as needed and as tolerated. Side effects of spironolactone are many and may include decreased libido, impotence and gynecomastia. Some patients with ascites secondary to cirrhosis may be managed without diuretic therapy by initiation of a diet including less than 2 grams of sodium per day. Patients with ascites refractory to sodium restricted diets are often placed on combination therapy with both spironolactone and furosemide. Ascites that does not respond to dietary sodium restriction and dual therapy with spironolactone and furosemide is treated with therapeutic paracentesis. Hydrochlorothiazide (A) is a thiazide diuretic, used as first-line therapy in the management of benign essential hypertension. Though this patient may also have hypertension for which hydrochlorothiazide might be effective, it is not the first choice for the management of ascites secondary to portal hypertension as a result of cirrhosis. Lactulose (B), is a synthetic disaccharide containing glucose and fructose, used for the management of hepatic encephalopathy, a complication of cirrhosis. It works by reducing the level of ammonia and other cerebral toxins in the circulation. It does not have a role in reducing ascites. Omeprazole (C) is a proton-pump inhibitor most commonly used to treat gastro-esophageal reflux disease. It may be used in patients with cirrhosis who have developed the complication of peptic ulcer disease, or for such patients who have undergone sclerotherapy for esophageal varices which have developed as a complication of portal hypertension. It is not considered first-line therapy in the management of ascites.

Question: What is the diagnosis for an adolescent male with profuse, unilateral epistaxis and a nasal mass?

nswer: Juvenile nasopharyngeal angiofibroma. Rapid Review Epistaxis Most common source: Anterior bleeds: Kiesselbach's plexus Posterior bleeds: Sphenopalatine artery Treatment is: Anterior bleeding: direct pressure, packing, cautery Posterior bleeding: packing (foley, gauze pack, intranasal balloon device) Admit patients with posterior packing to a monitored bed

Which of the following is a risk factor for developing nephrolithiasis? Glomerulonephrosis Peptic ulcer disease Peripheral vascular disease Previous cholelithiasis

orrect Answer ( B ) Explanation: Most stones are composed of calcium oxalate, alone or in combination with calcium phosphate. Hyperexcretion of calcium is a major contributor of stone formation. Many conditions are associated with hypercalciuria. Patients with peptic ulcer disease tend to ingest large amounts of calcium with food, in addition to absorbed alkali sources and antacids. Glomerulonephrosis (A), peripheral vascular disease (C), and previous cholelithiasis (D) are not risk factors for developing nephrolithiasis. Other risk factors include Crohn's disease, milk-alkali syndrome, hyperparathyroidism, hyperuricosuria, sarcoidosis, recurrent UTI, renal tubular acidosis, gout, laxative abuse, family history, dehydration, hypervitaminosis D, and previous kidney stone.

Question: What is "Blackwater" fever?

Answer: "Blackwater" fever describes the presence of hemoglobinuria resulting from hemolytic anemia in malaria. Rapid Review Malaria P. falciparum (deadliest), P. ovale, P. vivax, P. malariae P. ovale, P. vivax: hepatic phase Anopheles mosquito Immigrant, traveler Irregular fevers, diaphoresis P. falciparum: cerebral malaria, Blackwater fever Uncomplicated, no resistance areas rx: chloroquine Complicated, P. falciparum rx: quinidine + doxycycline

Question: What is the significance of "cells" and "flare" seen on slit lamp examination?

Answer: "Cells" and "flare" represent deep inflammation of the eye and is typically seen in iritis. Rapid Review Bacterial Conjunctivitis Patient will be complaining of red/pink eye with discharge usually worse in the morning PE will show purulent (yellow) discharge and crusting Most commonly caused by Staph aureus, Strep pneumoniae, H.Flu. Contact lens wearers: Pseudomonas aeruginosa Treatment is topical antibiotic drops

Question: What is the most accurate test in the diagnosis of adrenal insufficiency?

Answer: A cortisol level. Rapid Review Hypoparathyroidism Causes: damage during thyroid surgery > autoimmune ↓ Ca, ↑ phosphorus Hypocalcemia sx

Question: What is Caplan's syndrome?

Answer: A patient, typically a miner, with rheumatoid arthritis who acquires any of the pneumoconioses. Rapid Review Pneumoconiosis Asbestosis: shipping, roofing, plumbing Berylliosis: aerospace, fluorescent bulbs Byssinosis: cotton Silicosis: foundries, sandblasting, mines Coal worker's lung: coal Siderosis: iron Stannosis: tin SOB + nonproductive cough + chronic hypoxia PFTs: reduced lung volumes CXR: interstitial fibrosis Asbestos: from the roof, but affects the base (lower lobes) Silica, coal: from the base (earth), but affect the roof (upper lobes) Cor pulmonale

Question: What adjunctive therapy has been used to treat the symptoms of acute pharyngitis?

Answer: A single dose of dexamethasone has been shown to reduce inflammation and pain. Rapid Review Pharyngitis Viral > bacterial Bacterial GAS (S. Pyogenes) Centor criteria: age, fever, cervial lymphadenopathy, tonsillar exudate, no cough Rx: PCN, erythromycin (PCN allergic), steroids Complications: rheumatic fever, PSGN, abscess

Question: What role does magnesium sulfate play in asthma exacerbations?

Answer: Acts by relaxing bronchial smooth muscle and is indicated in severe asthma exacerbations. It has been shown to improve airflow obstruction and decrease hospital admissions. Rapid Review Asthma Asthma: airway inflammation + bronchial hyperresponsiveness + reversible airflow obstruction PEF <50%: severe exacerbation Treatments: O2: maintain SpO2 > 88% ß-agonists: ↑ cAMP → bronchodilation Anticholinergics: ↓ bronchoconstriction Corticosteroids: ↓ inflammation, administer early Mg: severe exacerbations Non-invasive ventilation: ↓ work of breathing Mechanical ventilation: Objective: maximize expiratory time Low respiratory rate High inspiratory flow rate Maintain plateau pressure <30 cm H2O Permissive hypercapnia to avoid breath stacking

Question: Hypertriglyceridemia is commonly associated with which gastrointestinal disease?

Answer: Acute pancreatitis. Rapid Review Hypertrigliceridemia Xanthomas Tendinous xanthomas Corneal arcus Treatment: 150-199: lifestyle modifications 200-499: consider pharmacologic therapy for high-risk pts >500: pharmacologic therapy

Question: What is the 5 year survival rate of Acute myeloid leukemia (AML)?

Answer: Approximately 25%. Rapid Review Acute Myeloid Leukemia (AML) Patient will be an adult Complaining of fever, fatigue, anemia, easy bruising or bleeding, petechiae, bone and joint pain, and persistent or frequent infections PE will show hepatosplenomegaly Labs will show Auer rods, normocytic, normochromic anemia

Question: How long should treatment with an SSRI continue in patients with generalized anxiety disorder?

Answer: At least 12 months.

Question: What are the classic physical exam findings that may be seen in basilar skull fracture?

Answer: Battle's sign, raccoon eyes, hemotympanum, CSF otorrhea and rhinorrhea. Rapid Review Subdural Hematoma Patient will be elderly or alcoholic With a history of a fall or traumatic head injury Complaining of headache, mental status changes, seizures, or focal deficits Diagnosis is made by non-contrast CT, crescent-shaped hematoma Most commonly caused by rupture of the bridging veins Treatment is neurosurgical consultation

Question: What common neuromuscular blockade can be used to treat severe lichen simplex chronicus?

Answer: Botulism toxin injections. Rapid Review Lichen Simplex Chronicus Patient will be complaining of generalized pruritus and frequent scratching PE will show multiple linear excoriations and thickened skin Treatment is reducing pruritus and minimizing rubbing and scratching

Question: What is the most common autoimmune blistering disease in the elderly?

Answer: Bullous pemphigoid. Rapid Review Bullous Pemphigoid Patient will be older than 60 Complaining of intensely pruritic papules that became large, tense blisters/bullae PE will show tense and firm blisters that do not extend with lateral pressure (Nikolsky sign negative) Most commonly caused by chronic autoimmune blistering disease Treatment is corticosteroids and immunosuppressants

Question: What is the central nervous system consequence of rapid correction of sodium?

Answer: Central pontine myelinolysis (also called osmotic demyelinating syndrome). Rapid Review Hyponatremia Serum sodium <135 Severity depends on rate of development of symptoms Nausea, vomiting, paresthesias, altered mental status Central pontine myelinolysis if corrected too fast Hypertonic saline for symptomatic patients

Question: What is the first step in treatment of chemical contamination of the eyes?

Answer: Copious irrigation with normal saline should be utilized to flush the eye for 15-30 minutes.

Question: What are the complications of untreated cholesteatoma?

Answer: Erosion of the ossicles with subsequent deafness and vertigo, and facial nerve palsies. Rapid Review Acquired Cholesteatoma Patient will have a history of chronic ear infections or tympanostomy tubes Complaining of painless otorrhea PE will show yellow or white mass behind the tympanic membrane Treatment is tympanomastoid surgery

Question: What is a major complication of esophageal dilation?

Answer: Esophageal perforation.

Question: What is the most common cause of Cushing syndrome?

Answer: Exogenous steroid administration. Rapid Review Hyperaldosteronism ↑ Na + ↓ K + HTN 1º: adrenal adenoma (most common cause), ↓ renin 2º: ↓ renal perfusion → RAAS activation, ↑ renin Rx: spironolactone (+ surgery if 1º)

What test is helpful in ruling out the diagnosis of Systemic lupus erythematosus? Anti-nuclear antibody Anti-Sm antibody C-reactive protein (CRP) Erythrocyte sedimentation rate (ESR)

Correct Answer ( A ) Explanation: Antinuclear antibody (ANA) is highly sensitive as it is present in nearly all patients with systemic lupus erythematosus (SLE). SLE is a multisystem, autoimmune disorder. The disease results in organ inflammation, dysfunction and eventually destruction. Patients can present with nephritis, thrombosis, carditis, pneumonitis, pulmonary hypertension, cerebrovascular accident or infection resulting from chronic immunomodulatory medications. The workup of patients with signs or symptoms concerning for SLE involves a number of serum tests all of which are imperfect. However, nearly all patients with the disease will have antinuclear antibody present on testing. The absence of this antibody virtually rules the disease out. Unfortunately, it is also present in up to 50% of patients without SLE and a positive test does not clinch the diagnosis. Anti-Sm antibody (B) is a more specific antibody for SLE but it is not present in all patients with the disease. CRP (C) does not typically rise in response to increased disease activity and may be normal in 10-15% of patients. ESR (D) is frequently elevated in SLE but may be normal in up to 30% of patients.

A father brings his 2-week-old newborn to the ED after a gagging episode at home where the infant "turned blue." The newborn was sleeping in his father's arms when he started choking, turned blue, and went limp. The father turned the baby over, did 5 back blows, and performed CPR for 5 minutes until the newborn started crying. On exam, the newborn appears sleepy but is easily arousable. Vital signs are HR 160, RR 30, T 37.6°C, and pulse ox is 99% on room air. Which of the following is the next best step in management? Admit to hospital for further workup Endotracheal intubation Epinephrine (1:10 000) IV Send home with reassurance

Correct Answer ( A ) Explanation: Brief resolved unexplained events (BRUE) [formerly ALTE] is an unexpected frightening episode characterized by some combination of apnea, color change, muscle tone change, choking or gagging, and fear that the child has died. The etiology is identified in only 50% of cases. The management of BRUE is variable, and admission criteria may vary between institutions. However, most experts generally agree that any infant who required resuscitation should be admitted for further workup. BRUE is not a diagnosis but a description of an event. Underlying causes include CNS infection, seizure, gastroesophageal reflux, intracranial hemorrhage, botulism, airway obstruction, electrolyte abnormality, and sepsis. Apnea may be an indication for endotracheal intubation (B). In this case, the etiology of the BRUE is unclear; the newborn is breathing without difficulty. Therefore, although intubation may be considered electively for further testing, it is not needed in the current management. Epinephrine (C) is not indicated in this patient, who has normal spontaneous circulation and no signs of circulatory collapse. Epinephrine is used in cases of asystole, pulseless electrical activity, ventricular fibrillation, and pulseless ventricular tachycardia. Sending this patient home (D) after an BRUE that required resuscitation is not recommended. This patient requires admission for observation and a workup to try to identify an etiology. Although the link between BRUE and sudden infant death syndrome is not fully established, the recurrence rate for severe BRUE is 68%, with an overall risk of death <1%.

A patient of yours becomes acutely jaundiced. She has had no contact with blood, body fluids, medical specimens or needles. She is currently sexually inactive. Which of the following is she most likely infected with? Hepatitis A Hepatitis B Hepatitis C Hepatitis D

Correct Answer ( A ) Explanation: Fecal-oral transmission is the most common route of transmission of hepatitis A and E, with A transmission being high in day-care center outbreaks or with contaminated shellfish and water. Hepatitis E is common in travelers or inhabitants of India, Mexico, Africa and Southeast Asia. Parenteral transmission is the route for Hepatitis B, C and D infection. This includes blood contact with infected blood products, bodily fluids or needles. It also includes perinatal transmission, which is common with Hepatitis B virus. Hepatitis C transmission is more common in contact with infected blood products than through sexual contact. Hepatitis D is common in Africa and Eastern Europe. Hepatitis B, C and D (B, C & D) are transmitted only parenterally, not enterally. This patient gives no risk factors for parenteral transmission.

Which of the following laboratory findings would you expect to find in primary hyperaldosteronism? Hypokalemia Hyponatremia Low serum aldosterone Metabolic acidosis

Correct Answer ( A ) Explanation: Hyperaldosteronism can stem from a variety of sources and most commonly leads to secondary hypertension. The most common cause is due to a aldosterone-producing adenoma, also referred to as Conn disease. Hypertension and hypokalemia are classically found, although the potassium can be normal. The workup begins with checking plasma aldosterone concentration and plasma renin activity. Measure after a high salt-diet or salt supplementation for a week. Also review 24-hour aldosterone level. Laboratory findings include hypokalemia, hypernatremia, and metabolic alkalosis. In patients with hyperaldosteronism, a CT or MRI should be performed to search for an adrenal adenoma. Almost 20% of patients with primary aldosteronism have impaired glucose tolerance resulting from the inhibitory effect of hypokalemia on insulin action and secretion; however, diabetes mellitus is no more common than in the general population. Clinically, patients present from complications of hypertension and hypokalemia (weakness, ileus). Treatment is with high-dose spironolactone or eplerenone (aldosterone antagonist) to normalize potassium levels. Side effects include gynecomastia, rash, impotence, and GI discomfort. Unilateral adrenalectomy is also an option in patients with a single adenoma. Hypernatremia occurs, not hyponatremia (B). Metabolic alkalosis, not metabolic acidosis (D), occurs. Conn syndrome is associated with elevated aldosterone levels (C). Low aldosterone levels is associated with Addison disease.

At what age should the Haemophilus influenzae type b conjugate vaccine series begin to be administered? 12 months 2 months 6 months Birth

Correct Answer ( B ) Explanation: The Haemophilus influenzae type b (Hib) conjugate vaccine series should begin to be administered at 2 months of age. Depending on the brand of vaccine, additional vaccinations should continue at 4 months of age. An additional vaccine may be needed at 6 months of age followed by a booster at 12 months of age. Haemophilus influenzae is an important pathogen in causing epiglottitis, an inflammatory condition of the supraglottic region that can cause life-threatening airway obstruction. Before the development of the vaccine, epiglottitis was more common in children. Now it is most common in adults with a mean age of 46 years. The hepatitis B vaccine begins at birth (A) is currently the only vaccine recommended at birth. The ages of 6 months (C) and 12 months (D) are incorrect.

A 45-year-old woman presents to the Emergency Department with a fever, productive cough, and diarrhea. Her chest X-ray shows a patchy unilobar infiltrate and her sodium is 127 mmol/L. Which of the following organisms is most likely to be responsible for these symptoms? Legionella pneumophila Pseudomonas aeruginosa Staphylococcus aureus Streptococcus pneumoniae

Correct Answer ( A ) Explanation: Legionella pneumophila is the most likely organism and causes Legionnaires' disease. It is responsible for about 5% of all pneumonias. Common symptoms include a fever, cough, diarrhea, and confusion. Chest X-ray findings vary but often show a patchy unilobar infiltrate. Laboratory abnormalities are common with hyponatremia being the classic finding. First-line treatment includes either respiratory quinolones (e.g. levofloxacin) or macrolides (e.g. azithromycin). Streptococcus pneumoniae (C) is the most common cause of community-acquired pneumonia (CAP). It presents classically with fever, chills, productive cough, and pain and a lobar opacity on chest X-ray. First-line treatment for inpatient management is a third-generation cephalosporin (e.g. ceftriaxone or cefotaxime). Outpatient pneumococcal pneumonia can be treated with amoxicillin or a macrolide. Pseudomonas aeruginosa (B) is a common cause of hospital-acquired pneumonia. It should be considered in patients with cystic fibrosis, bronchiectasis, or an immunocompromised state. Treatment can include an antipseudomonal penicillin (e.g. piperacillin-tazobactam) or an antipseudomonal cephalosporins (e.g. cefepime). Staphylococcus aureus (D) is a common cause of healthcare-acquired pneumonia and is seen following influenza infections. Treatment includes either vancomycin or linezolid since methicillin resistance is common.

A 25-year-old woman, whose last menses was one week ago, presents to the emergency department complaining of severe lower abdominal pain. She has a 4-hour history of right lower quadrant abdominal pain, anorexia, and nausea. She says the pain initially began around the bellybutton and then moved to the right lower quadrant. Her temperature is 38.3°C (101°F). Palpation of the left lower quadrant produces pain in the right lower quadrant. Pelvic examination shows right adnexal tenderness. Which of the following is the most likely diagnosis? Appendicitis Ectopic pregnancy Ovarian torsion Pelvic inflammatory disease

Correct Answer ( A ) Explanation: Right lower quadrant abdominal pain, anorexia, and nausea most likely suggests acute appendicitis. Appendicitis, an inflammation of the vestigial vermiform appendix, is one of the most common causes of the acute abdomen and one of the most frequent indications for an emergent abdominal surgical procedure worldwide. In women, right adnexal area tenderness may be present on pelvic examination. Rovsing's sign refers to pain in the right lower quadrant with palpation of the left lower quadrant, and is usually seen in patients with acute appendicitis. Ectopic pregnancy (B) typically presents with first trimester vaginal bleeding, abdominal pain, and amenorrhea. Ovarian torsion (C) is the rotation of the ovary at its pedicle which can result in occlusion of the ovarian artery/vein. Ovarian torsion is a gynecologic emergency and typically presents with sudden onset of sharp unilateral lower abdominal pain, nausea, and vomiting. Pelvic inflammatory disease (D) is an infection of the upper portion of the female GU tract (eg, uterus, fallopian tubes, ovaries) and typically presents with lower abdominal pain, vaginal discharge, fever, and dysuria.

A 12-year-old girl presents to the clinic with one day of cough, fever to 102°F, and extreme fatigue. She is awake and alert but appears tired. Her oxygen saturation is 95 percent, and respiratory rate is 15 breaths per minute. Lung auscultation reveals fine crackles in the left upper lobe. Which of the following is treatment of choice? Ampicillin Azithromycin Cefotaxime Clindamycin

Correct Answer ( B ) Explanation: The above patient's constellation of fever, cough, and focal lung findings are concerning for community-acquired pneumonia. The onset of symptoms was acute and severe, and the lung findings are focal, which is suggestive of a typical bacterial etiology. S. pneumoniae is the most frequent cause of "typical" bacterial pneumonia in children of all ages. However, in otherwise healthy children five years and older with CAP who are not ill enough to require hospitalization, M. pneumoniae and C. pneumoniae are the most likely pathogens. A macrolide is the empirical treatment of choice for typical bacterial pneumonia in otherwise healthy children > 5 years old being treated in outpatient setting. Among the macrolide antibiotics, clarithromycin and azithromycin have a more convenient dosing schedule and fewer side effects than erythromycin, but erythromycin is less expensive. Macrolide antibiotics may provide coverage for S. pneumoniae, which is the most frequent typical bacterial pathogen for all age groups. However, approximately 40 to 50 percent of S. pneumoniae isolates are resistant to macrolides. Failure to respond to macrolide therapy may indicate the development of a complication, a macrolide-resistant pathogen, or the need to alter therapy to provide better pneumococcal coverage. Given the significant resistance of S. pneumoniae to macrolides, fluoroquinolones (e.g., levofloxacin, moxifloxacin) are another reasonable alternative for the outpatient treatment of CAP. In addition to their excellent Gram-negative spectrum, the fluoroquinolones are active against a number of the pathogens responsible for CAP, including beta-lactam-susceptible and non-susceptible S. pneumoniae, M. pneumoniae, and C. pneumoniae. However, S. pneumoniae resistant to levofloxacin has been identified. Ampicillin (B) is used in neonatal infections. Cefotaxime (C) is an intravenous, third-generation cephalosporin used for inpatient treatment of community-acquired pneumonia for children with non-type-1 hypersensitivity reactions to penicillin. As the child is not hypoxemic, in respiratory distress, or toxic-appearing, she does not require inpatient treatment of her pneumonia at this time. A third-generation cephalosporin should also be used in the setting of confirmed beta-lactamase positive bacteria, such as Haemophilus influenzae or Moraxella catarrhalis or in the setting of complicated pneumonia without an identified pathogen. Clindamycin (D) may be used for treatment of typical bacterial pneumonias for children with type-1-hypersensitivity reactions to penicillin or for children with suspected aspiration pneumonia. Aspiration pneumonia typically involves the right middle or lower lobes and is more common in children with swallow dysfunction, such as those with static encephalopathy.

A 3-day-old neonate is brought to clinic with copious eye discharge and eyelid swelling bilaterally. Her mother received minimal prenatal care and delivered the neonate at home. Which of the following is indicated? Ceftriaxone intravenous Gonococcal PCR of eye discharge Oral erythromycin Reassurance with outpatient ophthalmologic follow-up

Correct Answer ( A ) Explanation: The neonate has findings concerning for ophthalmia neonatorum, neonatal conjunctivitis caused by Neisseria gonorrhoeae. N.gonorrhoeae is typically acquired during vaginal delivery and results in ophthalmia neonatorum in 30-40% of neonates born to mothers with gonorrheal cervicitis. Although all mothers should be screened during pregnancy, the infection may be acquired between the time of testing and delivery. Additionally, mothers with inadequate prenatal care may not be tested. Symptoms of ophthalmia neonatorum begin with conjunctivitis and discharge typically begins in the first two to five days after birth. The mucopurulent discharge is typically copious, and the eyelids may be so swollen that the conjunctiva cannot be visualized. When the diagnosis is suspected, a conjunctival swab should be sent for culture. Evaluation for extension of infection merits blood and CSF cultures, as well as evaluation for co-infection with Chlamydia trachomatis, syphilis, and HIV. When ophthalmia neonatorum is suspected, ceftriaxone should be empirically administered intramuscularly or intravenously pending culture and pathogen sensitivities. Isolated eye involvement is treated with a single dose of ceftriaxone, whereas disseminated disease is treated with seven days of ceftriaxone or cefotaxime. Gonococcal PCR of eye discharge (B) is not currently indicated, as nucleic amplification tests have not yet been sufficiently validated for diagnosing gonococcal conjunctivitis. Culture remains the gold standard for diagnosis. Oral erythromycin (C) is the treatment of choice for neonatal conjunctivitis or pneumonia caused by Chlamydia trachomatis. Chlamydial conjunctivitis typically begins five to fourteen days after delivery and, in comparison to ophthalmia neonatorum, has a less dramatic onset. Reassurance (D) and anticipatory guidance are inappropriate in the setting of suspected ophthalmia neonatorum.

A 70-year-old man presents to the ED after a fall down twelve stairs at home. He complains of a headache. There is a large parietal scalp hematoma. His non-contrast computed tomography scan of the head is shown above. Which of the following is the most likely source of bleeding? Bridging veins Meningeal veins Middle meningeal artery Superior sagittal sinus

Correct Answer ( A ) Explanation: This patient's imaging and physical examination are consistent with a traumatic acute subdural hematoma. A subdural hematoma is a collection of blood between the dura and the arachnoid mater. Subdural hematomas occur when there is rupture of the bridging veins from movement of the brain relative to the skull, such as that seen with acceleration-deceleration injuries. Subdural hematomas occur most commonly in individuals with brain atrophy, such as alcoholics and elderly patients. This results in blood filling the potential space between the dura and arachnoid. Subdural hematomas may be acute, subacute, or chronic. Acute subdural hematomas are characteristically seen after a traumatic head injury. Patients typically present with headache, mental status changes, seizures, or focal deficits. Subdural hematoma is diagnosed on non-contrast CT of the head and appears as a crescent-shaped hematoma that may cross suture lines. Management of subdural hematomas includes neurosurgical consultation for possible surgical evacuation; however, small subdural hematomas may be closely observed with serial non-contrast CTs of the head. Indications for emergent surgery in a patient with a subdural hematoma include neurologic deterioration or > 5 mm midline shift on CT. Meningeal veins (B) course through the dura mater while bridging veins drain the underlying neural tissue and puncture through the dura mater, making these more prone to shearing and causing a subdural hematoma than meningeal veins. The middle meningeal artery (C) is the vessel most commonly injured in epidural hematomas. The superior sagittal sinus (D) is where the meningeal veins and bridging veins drain into, and is typically not involved in causing a subdural hematoma.

A 45-year old man with diabetes mellitus sees you for the first time. If the patient has not previously received it, which one of the following vaccines is recommended for him? Hepatitis A Hepatitis B Meningococcal Varicella zoster

Correct Answer ( B ) Explanation: According to the recommendations the hepatitis B vaccine should be administered to all previously unvaccinated adults aged 19-59 with diabetes mellitus, as soon as possible after the diagnosis of diabetes is made. Hepatitis B vaccine should also be given to patients whom are at high risk for sexually transmitted diseases, health care personnel, chronic liver disease and HIV infected patients. Two doses should be administered at least 6 months apart. Hepatitis A vaccine (A) is recommended for any person with chronic liver disease or traveling and working in endemic areas. This vaccine is not indicated for a patient with diabetes. Meningococcal vaccine (C) is given to adults with asplenia, freshman college students in dormitories or military personnel. Varicella zoster vaccine (D) is recommended for adults 60 years of age or older regardless of whether they report a prior episode of herpes zoster.

A 52-year-old man presents with two weeks of worsening fatigue and generalized weakness. Over the past week his gums have been bleeding when he brushes his teeth. Laboratory studies show a normocytic, normochromic anemia with Auer rods on peripheral smear. Platelets count is 63 x 109/L. What is the most likely diagnosis? Acute lymphocytic leukemia Acute myeloid leukemia Chronic lymphocytic leukemia Chronic myeloid leukemia

Correct Answer ( B ) Explanation: Acute myeloid leukemia (AML) is the most common leukemia in adults and accounts for 15-20% of childhood leukemias. The median age of diagnosis in adults is approximately 65 years while the incidence peaks in the first two years of life in children. It is characterized by a clonal proliferation of myeloid precursors cells resulting in a variable reduction in the production of other cells, including red blood cells, platelets, and mature granulocytes. Patients present with symptoms related to pancytopenia including pallor, fatigue, easy bruising, gingival bleeding, petechiae, and infection. Laboratory studies will show a normocytic normochromic anemia with Auer rods (pink rod-like granular structures in the cytoplasm that are pathognomonic for myeloblasts) on peripheral smear. The majority of patients will have a platelet count < 100 x 109/L while the leukocyte count may be low, normal, or elevated. Diagnosis is confirmed by bone marrow biopsy. Acute lymphocytic leukemia (A) is the most common leukemia in children. Presenting symptoms are nonspecific and include bone pain, fever, fatigue, and lymphadenopathy. Chronic lymphocytic leukemia (C) is a disease primarily of older adults with findings related to anemia and thrombocytopenia as well as lymphadenopathy and splenomegaly on examination. The absolute lymphocyte count is generally > 5000 cells/mm3. Chronic myeloid leukemia (D) is the least common leukemia diagnosed and results in a neutrophilic leukocytosis.

Which of the following is a common cause of non-traumatic avascular necrosis of the hip? Cholecystitis Chronic corticosteroid use Chronic marijuana use Iron deficiency anemia

Correct Answer ( B ) Explanation: Although a specific causative disorder is not identified in 20% of the cases, known atraumatic causes include chronic corticosteroid therapy, chronic alcoholism, hemoglobinopathy (e.g., sickle cell anemia), and chronic pancreatitis. When a patient has an increasingly painful hip, buttock, thigh, or knee and no history of recent trauma, avascular necrosis (AVN) of the femoral head should be considered. AVN is bilateral in 40 to 80% of patients. It is common in relatively young patients, the mean age at diagnosis is 38 years. AVN also is an emerging complication associated with human immunodeficiency virus (HIV) infection. It is unclear whether the virus itself or the treatments are the pathogenic agents. Chronic pancreatititis and alcoholism, not cholecystititis (A) or chronic marijuana use (C) have been linked to avascular necrosis. Sickle cell anemia and not iron deficiency anemia (D) can lead to non-traumatic avascular necrosis.

A 22-year-old man presents with an asthma exacerbation. Which of the following is true regarding treatment modalities? Anticholinergic agents promote bronchodilation by decreasing cyclic adenosine monophosphate Beta-2-agonists promote bronchodilation by increasing cyclic adenosine monophosphate The onset of action for beta-2-agonists is 10 minutes The peak effect of corticosteroids occurs within 3-5 hours

Correct Answer ( B ) Explanation: Asthma is responsible for nearly 2 million ED visits per year. The mainstay of treatment consists of inhaled beta-2-agonists, inhaled anticholinergics, and corticosteroids. The combination of these drugs results in decreased rates of hospitalization and improvements in pulmonary function in the first 90 minutes. Beta-2-agonists promote bronchodilation by increasing cyclic adenosine monophosphate (cAMP). Their primary effect is on small peripheral airways with onset of action in less than 5 minutes. Studies have shown that administration of beta-2-agonists via metered-dose inhaler with a spacer provides similar bronchodilation when compared with nebulization. While use of the MDI may be more economical and take less time, it does require more supervision to ensure adequate use. Anticholinergic agents promote bronchodilation by decreasing cyclic guanosine monophosphate (cGMP), not cyclic adenosine monophosphate (A). Their primary effect is on the larger airways. Onset of action is also longer than with beta-2-agonists, taking up to 1-2 hours for peak effect. Corticosteroids work by inhibiting recruitment of inflammatory cells. They are indicated in all cases of moderate to severe asthma and in those who demonstrated limited response to beta-2-agonists. Onset of action is 1-2 hours with peak effect in about 24 hours, not 3-5 hours (C). Many studies have shown that oral and intravenous steroids provide equal benefit. Onset of action for beta-2-agonists is less than 5 minutes, not 10 minutes (D).

Which of the following is the most effective initial treatment for a benign esophageal stricture? Balloon dilation Balloon dilation and proton pump inhibitor Esophageal stent Nissen fundoplication «

Correct Answer ( B ) Explanation: Balloon dilation and proton pump inhibitor is the most effective initial treatment for a benign esophageal stricture. An esophageal stricture is the narrowing of the esophagus caused by the buildup of acid and is one of the complications of gastrointestinal reflux disease. Symptoms include heartburn, dysphagia, coughing, and shortness of breath. Treatment is aimed toward decreasing the symptoms of dysphagia and preventing the stricture from reoccurring. Therefore, the most effective initial treatment is to dilate the stricture using either a balloon dilator or a mechanical dilator followed by prescribing a proton pump inhibitor to prevent reoccurrence of acid buildup in the esophagus. Administering a proton pump inhibitor does not decrease the need to achieve adequate dilation with the balloon, but it has been proven to help reduce the need for subsequent dilations. Balloon dilation (A) alone will achieve the goal of immediately decreasing symptoms of dysphagia after opening the stricture, but it is not the best initial treatment because it won't prevent the stricture from reoccurring. Studies show that mechanically dilating the area is just as much of an important step in the treatment of esophageal strictures as prescribing a proton pump inhibitor is in order to prevent a stricture from reforming. An esophageal stent (C) is a treatment option for a benign esophageal stricture, but it should be reserved for those with refractory disease. In patients who do not respond to an initial treatment of dilation and a proton pump inhibitor, placement of a stent at the site of the esophageal stricture for at least six weeks will help to remodel the scar tissue. Complications include migration of the stent. Nissen fundoplication (D) is a laparoscopic surgical procedure performed to treat gastroesophageal reflux disease and, in turn, treat those with refractory esophageal strictures. In this procedure, the fundus is wrapped around the lower end of the esophagus, reinforcing the closure of the lower esophageal sphincter. This helps to close off the esophagus when the stomach contracts, therefore decreasing symptoms of gastroesophageal reflux disease. A surgical intervention, such as a Nissen fundoplication, has been reserved for those whose stricture cannot be dilated or those who are refractory to other means of decreasing acid buildup.

A 65-year-old woman presents with skin lesions seen above. When you apply lateral pressure to the bullae, they do not extend. Which of the following statements is correct regarding this condition? Bullae evolve into painful ulcers It is a chronic autoimmune disease Often seen in young individuals Oral lesions are present in a majority of cases

Correct Answer ( B ) Explanation: Bullous pemphigoid is a chronic autoimmune blistering disease often seen in patients older than 60 years of age. The blisters occur deep, within the epidermal basement membrane, which is why they do not extend with lateral pressure (Nikolsky sign negative). The Nikolsky sign is dislodgement of intact superficial epidermis by a shearing force, indicating a plane of cleavage in the skin. The defect may be due to staphylococcal toxin as in staph-scalded skin syndrome, or to epidermal antibodies as in pemphigus.The bullae evolve over weeks to months and most commonly occur in the axillae, abdomen, inner thighs, flexural forearms, and lower legs. The affected skin may be intensely pruritic. Pemphigus vulgaris (A and D) is associated with bullous lesions that often occur in the perioral region and erode to leave painful ulcers. Pemphigus vulgaris is associated with a positive Nikolsky sign. Bullous pemphigoid occurs mainly in individuals older (C) than 60 years of age. Pemphigus vulgaris occurs most commonly in younger individuals aged 40 to 60 years.

A 17-year-old student presents with unilateral hearing impairment. Weber and Rhinne testing of this afebrile patient supports conductive hearing loss of the left ear. Otoscopic examination reveals a waxy-appearing polyp situated behind a normal appearing tympanic membrane. There is no purulent discharge or tympanic perforation. Which of the following is the most likely diagnosis? Acoustic neuroma Cholesteatoma Otitis media Squamous cell carcinoma

Correct Answer ( B ) Explanation: Cholesteatoma is a mass of keratinized squamous epithelium that occurs in the middle ear or mastoid process occurring most frequently in teenagers. This ear-related metaplasia is not associated with cholesterol or gallbladder problems, as the name suggests. There are two types of cholesteatoma. The congenital type is less common and occurs medial to the tympanic membrane. The acquired type is more common and grows from the tympanic membrane. Conductive hearing impairment prevails, with imbalance and facial weakness being the most common associated symptoms. Examination may reveal inflammation, retrotympanic waxy appearing polyps or white-pasty discharge from the tympanic membrane. As such, this can easily be confused with chronic, suppurative otitis media. Microscopic excision surgery is required to prevent complications and to maintain or improve hearing. Acoustic neuromas (A) present with sensorineural, not conductive, hearing impairment. Furthermore, these masses occur in the inner ear and temporal bone on the vestibular nerve and would not likely be viewable during an otoscopic examination. Otitis media (C) presents with otalgia, hearing loss, fever and tympanic membrane abnormalities like erythema, bulging, or cloudiness. Although retrotympanic masses can result from chronic suppurative middle ear infections, current infection is unlikely in the above patient. Squamous cell carcinoma (D) is an epithelial cell malignancy that typically occurs on the external, not middle, ear due to prolonged sun exposure. It has an ulcerated, erythematous appearance with or without bleeding.

The treatment for celiac disease is a gluten free diet. Which of the following may be included in the diet of a patient with celiac disease? Barley Corn Rye Wheat

Correct Answer ( B ) Explanation: Corn is usually well tolerated by patients with celiac disease. Celiac disease is an inflammatory immune disorder that can affect any part of the intestines that is triggered by the gliadin component of gluten. It is precipitated by the ingestion of wheat, rye, and barley in individuals with certain genetic predispositions. Screening studies for the antiendomysial (EMA) and anti-tissue transglutaminase (anti-tTG) antibodies that are associated with celiac disease suggest a prevalence in Caucasian populations of about 1%. Once considered a disease of infancy, it now presents more often at 10-40 years of age, presumably because of longer breast-feeding periods and later introduction of gluten into the diet. High-risk groups for celiac disease include first-degree relatives and individuals with type 1 diabetes mellitus, autoimmune thyroid disease, primary biliary cirrhosis, Turner's syndrome, or Down syndrome. About 20% of patients diagnosed with irritable bowel syndrome or with microscopic (lymphocytic) colitis have celiac disease. Spontaneous remissions and exacerbations are common. Clinical presentation can range from significant disease causing diarrhea, steatorrhea, and weight loss to the absence of symptoms except those of a single nutrient deficiency (eg anemia, metabolic bone disease). Barley (A), rye (C), and wheat (D) should be excluded from the diet in those with celiac disease.

A 68-year-old woman presents with difficulty walking. Her neurologic examination is notable for ataxia. An MRI of the brain does not show any acute pathology. On laboratory analysis her sodium is 108 mEq/L. Which of the following is the most appropriate next step in management? 0.9% NaCl 500 cc bolus 3% NaCl 100 mL bolus Free water restriction Mannitol

Correct Answer ( B ) Explanation: Hyponatremia is defined as a serum sodium level less than 135 mEq/L. There are multiple causes of hyponatremia and when developing a management plan it is critical to determine the underlying etiology. In general, the volume status of the patient helps identify the cause. The list of causes is different in cases of hypovolemic hyponatremia, hypervolemic hyponatremia and euvolemic hyponatremia. Treatment decisions are made based on the etiology, the patient's clinical presentation, and duration of illness. Sodium is typically corrected over 48 to 72 hours. Treatment typically consists of either fluid replacement or fluid restriction depending on the cause. In some cases of severe hypervolemia, loop diuretics may be added to the therapy. As the sodium drops, patients begin to develop symptoms including headache, nausea, vomiting, confusion, seizures, neurologic abnormalities and coma. The central nervous system effects are likely caused by some degree of cerebral edema resulting from osmotic fluid shifts. Additionally, aggressive treatment with correction of the serum sodium values too quickly may cause neurologic damage and cerebral edema. Patients with neurologic symptoms require treatment with hypertonic (3%) saline. This is typically administered as a 100 ml bolus over 10 minutes. Severe hyponatremia is defined as a level below 120 mEq/L and treatment is initiated at this level even in asymptomatic patients. A 0.9% NaCl 500 cc bolus (A) is indicated for patients with hypovolemic hyponatremia. These patients are often dehydrated secondary to fluid losses like sweating, vomiting, diarrhea or as a result of diuretic use. In patients who are hypovolemic, it is important to avoid hypotonic intravenous fluids which may exacerbate the hyponatremia. Free water restriction (C) is first line treatment for patients with euvolemic and hypervolemic hyponatremia. Euvolemic hyponatremia is caused by SIADH or psychogenic polydipsia. Mannitol (D) is an osmotic diuretic that is used to treat various conditions such as increased intracerebral pressure. It is not used to treat hyponatremia.

A 36-year-old woman presents with numbness to her arms and legs for 2 hours. She had a total thyroidectomy 3 days prior to presentation. Vital signs are normal and the patient's exam is significant for a clean, well-healing surgical incision on her neck and twitching of the corners of the mouth with tapping at the angle of the jaw. What treatment is indicated? Obtain a CT scan of the head Replete calcium Replete potassium Stop thyroid hormone

Correct Answer ( B ) Explanation: Hypoparathyroidism is a common cause of hypocalcemia. During thyroidectomy, parathyroid glands are often removed and this may lead to hypocalcemia presenting as parasthesias. This patient also presents with a positive Chvostek's sign (twitching of the facial muscles in response to stimulation of the facial nerve) indicating hypocalcemia. Hypocalcemia can present with numerous clinical manifestations including muscle cramping, paresthesias, shortness of breath from bronchospasm and even hypotension, dysrhythmias and cardiovascular collapse in severe forms. QTc prolongation is common in moderate to severe hypocalcemia. Chvostek's and Trousseau's (carpal spasm with inflation of a blood pressure cuff) signs may be seen and indicate nerve irritability as a result of hypocalcemia. A serum calcium level is diagnostic and the patient should be treated with calcium repletion. This patient's symptoms are not the result of thyroid hormone (D) or hypokalemia (C). A head CT (A) is not indicated as the symptoms seen in this patient are not indicative of a central nervous system disorder.

A 26-year-old gravida 3 para 2 at 12 weeks gestation presents with fever, myalgias, headache, and malaise. There have been multiple cases of influenza in the community and her influenza swab is positive. Which one of the following is recommended by the Centers for Disease Control and Prevention in this situation? Acyclovir Oseltamivir Rimantadine Supportive therapy only

Correct Answer ( B ) Explanation: Influenza is primarily diagnosed clinically. Sudden onset of symptoms is a telltale sign of influenza. Common symptoms include high fever, headache, sore throat, myalgia, cough, rhinorrhea, and fatigue. The Centers for Disease Control and Prevention recommends treatment for persons at higher risk for complications from influenza because studies have shown that antiviral treatment may shorten the duration of symptoms and reduce the risk of complications, especially in these higher risk populations. Pregnant and postpartum women, including those who have had pregnancy loss, are at risk of severe influenza-related complications because of the changes in immune, respiratory, and cardiovascular systems that occur during pregnancy. The Center for Disease Control recommends that neuraminidase inhibitors be prescribed for pregnant women and for those up to two weeks postpartum who have suspected or confirmed influenza. Women can continue to breastfeed while being treated with antivirals. Treatment with antiviral agents, should be started within 48 hours of symptom onset. The recommended treatment of influenza in pregnancy is one of the neuraminidase inhibitors, such as oseltamivir. Neuraminidase inhibitors have modest effectiveness in reducing influenza-related symptoms in patients and they protect against both strains of influenza A and influenza B. Adamantanes, such as rimantadine (C) are active only against influenza A viruses and therefore are not recommended. Other antivirals such as acyclovir (A) and famciclovir are not active against influenza A and B. Supportive therapy (D) with over-the-counter antipyretics and anti-inflammatory agents can be used for symptomatic relief in patients with influenza as an adjunct to neuraminidase inhibitors.

A 26-year-old woman is brought in by her husband for evaluation. He states that over the last week, she has exhibited a number of concerning symptoms including tremors and seizures. On entering the room, the patient is noted to be resting comfortably. When asked about the tremor, the patient begins to exhibit coarse diffuse tremors. During the evaluation the patient has generalized shaking but is able to respond to questions. Further discussion with the patient's husband reveals that they are currently going through a separation and there has been a lot of stress at home. Which of the following disorders accounts for the patient's symptoms? Conversion disorder Factitious disorder Hypochondriasis Somatization

Correct Answer ( B ) Explanation: This patient exhibits signs and symptoms of factitious disorder. Factitious disorder is characterized by falsified general medical or psychiatric symptoms. Patients deceptively misrepresent, simulate, or cause symptoms of an illness or injury in themselves, even in the absence of obvious external rewards such as financial gain, housing, or medications. Symptoms may develop after an identifiable psychosocial stress or as part of a pattern of general life (i.e. this is the way the patient deals with life events). Symptoms can be both psychological and physical. If the patient admits to producing the symptoms, they would not be included in the factitious category. This patient presents with volitional tremors and pseudoseizures that are characterized by general shaking but preserved cognitive function. Conversion disorder (A) describes the presence of a single unexplained symptom affecting voluntary or sensory function suggestive of a neurologic or medical condition. Patients are typically oblivious to or express no concern about the symptom. Hypochondriasis (C) is a preoccupation with having a serious medical condition. Somatization (D) is the unexplained presence of physical symptoms (multiple) that result in seeking treatment and significant social impairment.

Which of the following patients requires radiologic imaging for her back pain? A 26-year-old woman with unilateral costovertebral angle tenderness, fever, and dysuria A 30-year-old woman with midline lumbar pain and tenderness, fever, and difficulty urinating A 35-year-old woman with bilateral lumbar level back pain after lifting furniture that is worse with movement and alleviated with rest A 40-year-old woman with lumbar-level pain and tenderness that radiates to her left anterior thigh, with a normal neurologic exam

Correct Answer ( B ) Explanation: This patient has symptoms suspicious for a spinal epidural abscess. The classic triad is severe back pain, fever, and neurological deficits. In her case, urinary retention is her deficit, which by itself is worrisome for cauda equina syndrome. The diagnostic test of choice is MRI. Epidural abscess may require evacuation by a spine surgeon. Patients are often on IV antibiotics for many weeks. However, to avoid sterilization of the abscess and facilitate isolation of the causative organism, it is recommended that antibiotic therapy be withheld in the emergency department for suspected (or even radiographically confirmed) cases. Patients at risk for spinal epidural abscess include intravenous drug users, immunocompromised patients, alcohol abusers, diabetics, patients with malignancy, and those with recent spine procedures. Pyelonephritis (A) is an infection of the renal parenchyma and upper collecting system. In an otherwise healthy, young patient, treatment can be performed on an outpatient basis with current guidelines recommending 14 days of antibiotics. Radiographic imaging is not needed unless there is suspicion of an infected kidney stone. Nonspecific back pain (C) in the absence of "red flags" (e.g., fever, neurologic deficit, IV drug use, trauma) does not require radiographic imaging and can be managed symptomatically with resumption of usual daily activities as soon as possible. Bed rest has been shown to prolong the duration of pain, as have back exercises. Most nonspecific back pain resolves in 4-6 weeks. Radiculopathy (D) or nerve impingement syndrome may be caused by a disk herniation or other irritation of the nerve root. Given the location of symptoms, compression of the L4-L5 nerve root should be suspected but without worrisome features; imaging in the emergency department is not indicated. If her symptoms do not resolve in 4-6 weeks, an MRI of the lumbosacral spine may be warranted.

Which of the following medications is most likely to cause side effects such as tardive dyskinesia and have limited efficacy in treating the negative symptoms of schizophrenia? Aripiprazol Haloperidol Quetiapine Risperidone

Correct Answer ( B ) Explanation: Typical antipsychotics such as haloperidol have a relatively high risk of adverse effects such as tardive dyskinesia and parkinsonism, especially in older patients. They are mainly effective against the positive symptoms. Antipsychotics are never to be given as an isolated measure; other forms of treatment, such as individual or family therapy and psychosocial measures, should continue. Special education and support measures (compliance therapy) are useful where there are compliance problems. Schizophrenia is a chronic psychiatric disorder with no single feature that is pathognomic. Without treatment many patients may experience a downward drift in socioeconomic class. Atypical antipsychotics are associated with fewer extrapyramidal side effects and also can treat the negative symptoms of schizophrenia. Aripiprazole (A), quetiapine (C), and risperidone (D), are all atypical antipsychotics.

A 28-year-old man reports to the emergency department with acute 10/10 flank pain, dysuria and vomiting. Urinalysis reveals large amounts of blood and pus. Serum laboratory testing shows electrolyte alterations. Which of the following is the most appropriate initial diagnostic test? KUB radiograph Noncontrast computed tomography Percutaneous nephrostomy Renal magnetic resonance imaging

Correct Answer ( B ) Explanation: Urolithiasis refers to the presence or formation of urinary stones anywhere in the urinary system. More specifically, nephrolithiasis refers to stones which arise in the kidney, ureterolithiasis refers to stones in the ureters, and cystolithiasis refers to stones which have passed into or form within the urinary bladder. Urolithiasis arises from accretion of hard, solid nonmetallic minerals in the urine, usually due to supersaturation (increased secretion) of stone-forming compounds, chemical and physical stimuli which predispose stone formation, vitamin A and C deficiency, urostasis and inadequate urinary amounts of magnesium or citrate. Stones are commonly composed of calcium-oxalate, uric acid, cysteine and struvite (magnesium ammonium phosphate). 70-90% of all stones are calcium oxalate or calcium phosphate, and are visible on radiography. 5-10% of all stones are composed of uric acid, which are radiolucent and not readily visible on radiography. Risk factors for hyperoxaluria include Crohn's disease, gastric bypass or ilieal disease. Often, stones are passed asymptomatically. However, if a stone grows to over 3-5mm, it typically will cause symptoms as it lodges in the ureters and causes ureteral spasm. This is known as renal colic, and is characterized by waves of flank and back pain which radiate to the groin and genitals, nausea, and vomiting. An obstructing stone can lead to infection and patients develop fever and pyuria. This is a urologic emergency. To confirm the diagnosis, a urinalysis will reveal infection and a noncontrast CT scan of the abdomen and pelvis will show a stone with associated hydroureter or hydronephrosis. Patients should be treated with antibiotics and analgesics and consultation with an urologist should be obtained immediately. A KUB radiograph (A) will miss many stones, particularly uric acid stones, as such, CT is the better imaging modality. Percutaneous nephrostomy (C) is one of many urinary system decompression procedures used in the treatment, not diagnosis, of urolithiasis. Renal magnetic resonance imaging (D) is not the initial diagnostic test used in the evaluation of renal colic. It is time consuming and costly.

Which of the following is considered appropriate treatment for bacterial vaginosis? Azithromycin 1 g PO x 1 Ceftriaxone 250 mg IM x 1 Clindamycin 300 mg PO BID x 7 days Metronidazole 2 g PO x 1

Correct Answer ( C ) Explanation: Bacterial vaginosis (BV) is a polymicrobial vaginal infection that occurs when the normal Lactobacillus species are replaced with high concentrations of anaerobic bacteria. BV is characterized by a thin, white discharge that has a fishy odor, a vaginal pH of >4.5 and clue cells on microscopy. It can be associated with sexual intercourse or anything that disrupts the normal vaginal flora (ie, douching). It is not necessarily a sexually transmitted infection. The CDC recommended treatment regimens include metronidazole 500 mg PO BID for 7 days, metronidazole gel 0.75% 5 g intravaginally daily for 5 days, or clindamycin cream 2% 5 g intravaginally at bedtime for 7 days. For patients who cannot tolerate metronidazole, clindamycin 300 mg PO BID for 7 days is an alternative. Alternative treatment options according to the CDC include tinidazole 2 g PO daily for 2 days, tinidazole 1 g PO daily for 5 days, or clindamycin ovules 100 mg intravaginally daily for 3 days. Azithromycin 1 g PO (A) is for treatment of uncomplicated Chlamydia cervicitis/urethritis. Ceftriaxone 250 mg IM (B) is treatment for Gonorrhea cervicitis/urethritis. Metronidazole 2 g PO (D) is therapy for trichomonas vaginitis. Although metronidazole is an appropriate medication for treatment of BV, the one time dosing is not sufficient.

Which of the following is true regarding anorexia nervosa? Affects men more than women Anti-depressants are an effective treatment Associated with a body image disturbance Menorrhagia is an early symptom

Correct Answer ( C ) Explanation: Eating disorders are the third leading chronic illness, after obesity and asthma in adolescent girls. An individual with anorexia nervosa refuses to maintain a minimally normal body weight, is fearful of gaining weight, and exhibits a distorted body self-image. The patient's body image is the predominant measure of self-worth, along with denial of the seriousness of the illness. The long-term mortality rate for anorexia nervosa is 6% to 20%, the highest rate for any psychiatric disorder. Anorexia is associated with amenorrhea, depression, fatigue, weakness, hair loss, bone pain, constipation, and abdominal pain. Signs include brittle hair and nails, dry, scaly skin, loss of subcutaneous fat, fine facial and body hair (lanugo), and breast and vaginal atrophy. A prime objective in assessment is to distinguish "normal dieters" from individuals with eating disorders. Another important aspect to evaluation is to exclude certain medical conditions such as inflammatory bowel disease, hyperthyroidism, chronic infection, diabetes mellitus, and Addison's disease. Indications for inpatient management include extremely low weight (<75% of expected body weight) or rapid weight loss; severe electrolyte imbalances, cardiac disturbances, or other acute medical disorders; severe or intractable purging; psychosis or a high risk of suicide; and symptoms refractory to outpatient treatment. A multidisciplinary team that includes a primary physician, mental health professional, and nutritionist should manage patients with eating disorders. Refeeding should occur in a monitored setting due to the risk of dysrhythmia. Various antidepressants (B) are effective for treatment of bulimia nervosa but have not shown definite benefit for anorexia nervosa. Cognitive-behavioral therapy has been shown to be the most effective psychological approach to anorexia nervosa. Girls who are postmenarchal are typically amenorrheic (D) and do not experience menorrhagia. Almost 95% of those with anorexia are women (A).

A 4-year-old boy presents to the clinic for a well child visit. His mother reports he has frequent nosebleeds that last for several minutes at a time. During an episode, she has the boy hold pressure with a tissue. She denies any trauma to the nose. On examination, his nasal mucosa is dry, without polyps or foreign bodies. Which of the following is the best advice to give this mother during episodes of epistaxis? Administer local oxymetazoline at the onset of symptoms Apply pressure to the nares with the child lying supine Compress the nares with the child in an upright position and head tilted forward Perform anterior nasal packing for ten minutes

Correct Answer ( C ) Explanation: Epistaxis is common in childhood, and usually stops spontaneously within a few minutes. Often, epistaxis presents with slow, free flowing blood from one nostril. If blood is swallowed, the child may also present with blood in the stool or blood in emesis. The common causes of epistaxis include digital trauma, foreign body, dry air, and inflammation, as well as nasal polyps or growths. Rare but more serious causes of epistaxis are hereditary hemorrhagic telangiectasia, hemangiomas, clotting factor deficiencies, hypertension, renal failure, and vascular congestion. The best way to stop the bleeding is to keep the child quiet, sit him in an upright position, and compress the nares with the head tilted slightly forward. Cold compresses can also help staunch bleeding. Preventive measures include a humidifier, nasal saline drops, and petroleum to the nasal septum. In more severe cases, treatment may include topical silver nitrate. Patients with severe epistaxis should be monitored for anemia and the need for blood transfusions. Oxymetazoline (A) is a topical decongestant that acts as an alpha-1 agonist and partial alpha-2 agonist to vasoconstrict the vessels. This can improve nasal congestion or epistaxis. However, the first line treatment of epistaxis should be to hold pressure. Oxymetazoline should not be used for greater than three days due to rhinitis medicamentosa, or rebound congestion. The child should not be supine or tilt the head back (B), as this can cause the child to swallow blood. Anterior nasal packing (D) can be used in persistent epistaxis that does not respond to compressing the nares or applications of oxymetazoline.

A 38-year-old woman presents to your office with complaints of wrist pain and swelling. Physical exam findings include a flesh-colored, smooth, firm, rounded swelling on the dorsal aspect of her wrist. Which of the following is the most appropriate first step in management? Closed rupture by force Corticosteroid injection Cyst aspiration Referral for surgical intervention

Correct Answer ( C ) Explanation: Ganglion cysts are benign lesions and are one of the most common soft tissue swellings of the wrist and hand. Ganglion cysts are thought to be caused by a degeneration of the mucoid connective tissue, although a definitive etiology has yet to be established. They are most commonly seen at the dorsum of the wrist at the scapholunate joint, but may occur at any joint or tendon sheath. Patients may present with an obvious swelling or may have joint pain as the primary complaint without an obvious etiology. Initial treatment is with nonsurgical measures including observation or needle aspiration. Many patients experience spontaneous resolution of the ganglion cyst without intervention and recurrence is common regardless of intervention used. Historically, people believed that ganglion cysts could be cured by using force to cause a closed rupture (A), hence the expression "bashing it with a bible". This is not a recommended modality to treat ganglion cysts. Corticosteroid injection (B) should be avoided as it may cause thinning of the skin overlying the cyst. Referral for surgical intervention (D) may be appropriate after conservative measures such as observation and needle aspiration fail.

A 67-year-old woman presents to your office accompanied by her adult daughter with a complaint of anxiety. She says that she's always been a "worrier," but since the birth of her grandchild last year her anxiety has gotten worse. Further discussion reveals that she sleeps only 2-3 hours per night, has daily headaches, and her daughter complains that she calls their house at least 15 times every day asking about her grandchild's safety. Which of the following is the most appropriate therapy? Imipramine Lorazepam Paroxetine Quetiapine

Correct Answer ( C ) Explanation: Generalized anxiety disorder (GAD) is defined by having uncontrollable anxiety and worry that interfere with daily activities for at least 6 months. Symptoms occur on more days than not during this time period. First-line medications for GAD include SSRI's or SNRI's, including paroxetine. Psychotherapy is also recommended in the treatment of GAD. If one SSRI does not have the desired effect, then a trial of a different SSRI is used prior to initiating a second-line medication. Imipramine (A) is a tricyclic antidepressant that is considered second-line in the treatment of GAD. Lorazepam (B) is a benzodiazepine that can be used to augment SSRI therapy, especially in cases of agitation. Use of benzodiazepines is declining due to concerns about dependence and tolerance. Quetiapine (D) is an atypical antipsychotic that has been used in the treatment of GAD, however its use is off-label and it is not currently approved by the FDA.

An 18-year-old college student presents to the emergency department with headache, fever, and stiff neck for two days. He thought he had a cold and has been taking acetaminophen without relief of his headache. He has no medical history. His parents did not have him vaccinated because of worries about autism. Vital signs are T 38.8.°C, BP 110/50 mm Hg, HR 137/min, and RR 25/min. He is sleepy but arousable. When you flex the patient's head, you note flexion of the patient's lower extremities (hips and knees). Skin exam reveals diffuse petechiae. What is the most likely diagnosis and recommended therapy? Cryptococcus; amphotericin B and flucytosine Herpes simplex virus; acyclovir Neisseria meningitidis; ceftriaxone plus vancomycin Streptococcus pneumoniae; ceftriaxone plus vancomycin West Nile virus; supportive care

Correct Answer ( C ) Explanation: Neisseria meningitidis is a common cause of meningitis in crowded living conditions such as college dorms and military barracks. The classic signs of meningitis are fever, headache, neck stiffness, and mental status change. Petechiae and purpura may also be seen with more advanced cases of meningococcemia. N. meningitidis is an encapsulated bacterium that gains entry through the upper airway and spreads hematogenously to the subarachnoid space. Elements of the capsule trigger an inflammatory cascade, causing the brain and meninges to become edematous, leading to decreased CSF drainage. Disruption of the blood-brain barrier also allows entry of protein and water into the brain, resulting in vasogenic edema. The Brudzinski sign is positive if flexion of the neck results in passive flexion of the hips and knees. It is more sensitive than the Kernig sign. Diagnosis of meningitis is made with lumbar puncture (LP). The LP typically reveals an opening pressure > 30, WBC > 1000 with > 80% PMNs, glucose < 40 mg/dL, protein > 200 mg/dL, and cultures test positive for gram negative bacteria. Early treatment of meningitis is essential. Antibiotics should be administered as quickly as possible, even before lumbar puncture. Cryptococcal meningitis (A) is an infection of immunocompromised patients. It is considered an AIDS-defining illness but does not occur exclusively in patients with HIV. It may present with fever, headache, mental status changes, and focal neurologic deficits. Meningismus is uncommon. If cryptococcal infection is suspected, be sure to check an opening pressure on LP. Cryptococcus in the CSF is diagnosed by positive cryptococcal antigen, positive culture, or positive staining with india ink. Treatment is with amphotericin B and flucytosine. Herpes simplex virus (B) is an uncommon cause of viral meningitis in patients with an intact immune system. Most viral meningitides are caused by enteroviruses (echovirus, coxsackievirus, enterovirus). Streptococcus pneumoniae (D) is the most common cause of bacterial meningitis in adults. It is an encapsulated organism that enters the body via the upper respiratory tract. It has many of the same features as N. meningitis, but it is not associated with dormitory living and does not result in the purpura of fulminant meningococcemia. Treatment is with ceftriaxone and vancomycin. West Nile virus (E) causes encephalitis. It is an arbovirus, transmitted by mosquito bite. The virus replicates outside the brain and crosses the blood-brain barrier during viremia. It primarily affects the gray matter, resulting in cognitive dysfunction, psychiatric abnormalities, lethargy, and seizures. Headache and fever are usually present. Treatment is mainly supportive.

An 80-year-old homeless man is found in his riverside tent with confusion and fever in July. On physical examination, the patient is disoriented with T 38.2°C, HR 122 bpm, BP 106/84 mm Hg, and oxygen saturation 99% on room air. You note splenomegaly, flaccid paralysis, and multiple mosquito bites. Which lab abnormality would lead you to suspect West Nile encephalitis in this patient? Decreased CSF glucose level Leukocytosis Lymphopenia Markedly increased serum ALT/AST

Correct Answer ( C ) Explanation: Patients with West Nile encephalitis (WNE) will have leukopenia with a pronounced and prolonged lymphopenia, which can aid in distinguishing it from other causes of encephalitis. WNE is an arthropodborne virus (arbovirus) endemic to the Middle East and now found throughout the United States. It is transmitted by the Aedes mosquito, with birds serving as the intermediate host. Most people with WNE will remain asymptomatic or have a mild viral syndrome. Patients with encephalitis will present with new psychiatric symptoms, cognitive deficits, seizures, flaccid paralysis, and tremors. Most will have a headache and low-grade fever. In patients with meningoencephalitis, a lumbar puncture will show a pleocytosis with mostly lymphocytes, normal to elevated glucose, and increased protein. Decreased CSF glucose (A) is found in patients with bacterial or fungal meningitis. It is generally normal in viral meningitides. A mildly increased ALT/AST (D) may be seen in WNE, Epstein-Barr virus, Rocky Mountain spotted fever, and ehrlichiosis; however, markedly elevated transaminases are not generally part of arboviral infection. Leukocytosis (B) in a patient with WNE suggests an overlying bacterial infection.

A 24-year-old pregnant woman presents with a rash on her hands with itching that is worse at night. Physical exam shows excoriations over the web spaces of the hands with tiny burrows noted. Several members in her household have similar symptoms. Which of the following is the most appropriate treatment? Ivermectin Lindane 1% cream Permethrin 5% cream Sulfur in petroleum 5%

Correct Answer ( C ) Explanation: Permethrin 5% cream is a safe treatment for scabies during pregnancy. Less than 2% of an applied dose is absorbed systemically although residual effects of the drug remain for up to ten days after application. Scabies is caused by a mite infestation with Sarcoptes scabiei. These mites burrow under the skin of infested individuals leaving behind eggs and feces in the small epidermal tunnels that they create. These burrows are usually found in the web spaces between digits, intertriginous areas, and flexor creases. Symptoms include itching that is worse at night and a rash or itching that is worse in web spaces or intertriginous areas. History will often reveal others in the household with similar symptoms. Diagnosis of scabies is usually clinical, however, skin scrapings can reveal microscopic confirmation of adult mites or their eggs. Treatment includes permethrin 5% cream, lindane 1% cream (not available in the USA due to side effects), ivermectin, or crotamiton 10% cream. Permethrin 5% cream is safe during pregnancy and is applied over the entire body from the neck down and left on for eight hours. Afterwards, it is washed off entirely. Patients should also be instructed to launder all clothing and bed linen in hot water to prevent reinfestation. Ivermectin (A) should not be used during pregnancy and is not safe to use in children less than five years. It is a pregnancy category C drug and has shown to be teratogenic in studies using mice, rats, and rabbits. Lindane 1% cream (B) is a pregnancy category C drug and has been shown to be absorbed systemically in studies. There are concerns regarding neurotoxicity and hematotoxicity with its use making it an unsuitable choice for pregnant females or young children. Lindane is no longer available in the USA. Sulfur in petroleum 5% (D) is safe during pregnancy, however, the unpleasant odor and staining associated with this drug make it a less attractive choice for treatment.

You are treating a 50-year-old coal-miner's hypertension with lisinopril. He has been complaining of 3-months of progressive dyspnea. You order a chest radiograph which shows bilateral upper lobe honeycombing. A high resolution computed tomogram shows multiple small, round opacities only in the upper lobes. The lower lobes appear normal. Which of the following is the most likely diagnosis? Goodpasture's syndrome Granulomatosis with polyangiitis (GPA) Pneumoconiosis Scleroderma

Correct Answer ( C ) Explanation: Pneumoconiosis is an occupational respiratory disease due to inhalation of inorganic dusts. It is common in miners. In 2010, it resulted in 125,000 deaths in the US. History is key in determining the probable agent. A common form is Coalworker's pneumoconiosis, also known as miner's lung or black lung, and which is due to chronic inhalation of coal and carbon particles. Machinists, especially grinders, and pottery makers, can develop the silicon particle form called silicosis. Construction workers, especially boat builders, can suffer from asbestosis. Clinically, the diagnosis is made in a patient with dyspnea and the classic occupational exposures as above. Radiography may reveal small cystic radiolucencies described as honeycombing. High-resolution computed tomography offers better evaluation, and typically shows small, round opacities which denote inflammatory areas of dust-laden macrophages and fibrosis, as is quite common in the carbon deposits of coalworker's pneumoconiosis. Interestingly, the upper lobes are more affected in miner's lung and silicosis, while the lower lobes are more affected in asbestosis. Progressive pulmonary fibrosis, with resultant restrictive lung disease pathology and pulmonary function test results, typically occurs. Other than removing oneself from the offending agent and dusty environment, treatment is mainly supportive. Goodpasture's syndrome (A) is an interstitial lung disease characterized by alveolar hemorrhage and glomerulonephritis. Scleroderma (D) is a collagen vascular disease whose pulmonary findings include lower lobe fibrosis and pulmonary hypertension. Upper lobe pathology is uncommon. Granulomatosis with polyangiitis (GPA) (B) is a vasculitic interstitial lung disease whose imaging reveals necrotizing granulomas.

A 29-year-old woman presents with a chief complaint of dry eyes. Which of the following would indicate the need for testing for a systemic illness? Anticholinergic use Clear discharge Dry mouth Long term contact lens use

Correct Answer ( C ) Explanation: Sjogren's syndrome is a systemic autoimmune disease in which immune cells attack and destroy the exocrine glands that produce tears and saliva leading to dry mouth and eyes (keratoconjunctivitis sicca). Work up for Sjogren's includes lab work to identify the auto-antibodies (ANA, RF, anti SS-A, anti SS-B (formally anti-Ro and anti-La)), a Schirmer test (for tear production) and a biopsy of the salivary glands. Patients may also present with dryness of other mucous membranes including the vagina and the nasal passageways. Complications of Sjogren's include dysparenuria, corneal abrasions, dental caries, malabsorption syndromes to due to pancreatic failure, lymphoma, polyneuropathy, lung and kidney disease and other rheumatologial complaints. Treatment options include artificial tears and saliva, topical ophthalmic steroids, systemic steroids or cytoxic agents or a combination of both. Anticholinergic use (A) can lead to dry eyes, but symptoms will resolve when the agent is withdrawn. Long term contact lens use (D) can increase tear evaporation, decrease tear flow and increase infections, but is not due to a systemic illness. Clear discharge (B) is associated with viral conjunctivitis and is typically self-limited.

A 7-year-old male presents to the Emergency Department with an ankle injury. He fell while on equipment at a nearby playground. There is pain, swelling, and ecchymosis over the medial malleolus. An ankle X-ray is obtained and is shown above. Which of the following is the most likely diagnosis? Greenstick fracture Salter-Harris type II fracture Salter-Harris type III fracture Torus fracture

Correct Answer ( C ) Explanation: The Salter-Harris classification is used to describe pediatric injuries involving the physis, the cartilaginous epiphyseal growth plate at the end of long bones. There are five types of fractures in this classification, and management depends on the type and severity of the fracture. Salter-Harris type III fractures are intra-articular injuries that involve the physis and the epiphysis. These are often unstable fractures and are managed based on size and degree of displacement. Consultation with an orthopedic surgeon is recommended. A greenstick fracture (A) is an incomplete long bone fracture pattern that involves bending along one side of the bony cortex. A Salter-Harris type II fracture involves the physis and the metaphysis (shaft). This is the most common type of Salter-Harris fracture and is usually stable. It carries an excellent prognosis and usually managed nonoperatively. A torus fracture (D) is an incomplete fracture caused by compressive forces that creates periosteal bulging or buckling.

A five-year-old boy is brought to the emergency room for altered mental status. The family had a party at their house the night before. In the morning, they found their son on the floor surrounded by empty cans of beer. He appears sleepy, and on the way to the hospital, he vomited twice. On examination, the boy is sedated with sluggish pupils and flushed skin. Which of the following abnormalities would you expect with the boy's toxic ingestion? Abnormal head computed tomography (CT) Hyperkalemia Hypoglycemia Metabolic alkalosis

Correct Answer ( C ) Explanation: The signs and symptoms of the boy in the vignette are suspicious for ethanol ingestion. Children younger than six years of age often ingest ethanol when their exploratory behaviors lead them to unattended alcoholic beverages or to unsecured household products with high ethanol concentrations. Ethanol acts as a CNS sedative in a dose-dependent manner in overdose. Ethanol intoxication commonly manifests as altered behavior, lethargy, coma, ataxia, slurred speech, hypothermia, bradycardia, hypotension, and respiratory depression. A characteristic sickly sweet breath odor is often prominent. A history of ingestion from the caregiver or patient and characteristic physical findings are sufficient to establish the diagnosis of ethanol intoxication. Rapid blood glucose measurements should be performed in all patients with altered mental status after ethanol ingestion. Decreased blood glucose below 40 mg/dL is well described in children who have ingested ethanol. This ethanol-induced hypoglycemia results from exhausted glycogen reserves. Abnormal head CT (A) is not an expected finding in ethanol ingestion. All children with altered mental status should undergo an assessment for signs of head trauma. But in this vignette, there are no signs of head trauma that would make you suspect a traumatic brain injury. Hyperkalemia (B) and metabolic alkalosis (D) are not expected findings in ethanol ingestion. Electrolytes may be altered by protracted vomiting (more than the only two episodes experienced here) and hypovolemia associated with acute ethanol ingestion and associated laboratory findings may include hypokalemia and metabolic alkalosis.

A 15-year-old girl presents with fever after returning from a trip to Africa. She states that she has been getting fevers every 3 days for the last 10 days. What test will confirm a diagnosis of malaria? Complete blood count Enzyme linked Immunosorbent assay (ELISA) Thick and thin smears VDRL

Correct Answer ( C ) Explanation: Thick and thin smears of a patient's blood sample are the gold standard for the diagnosis of malaria. Malaria is an acute febrile illness caused by 4 major types of Plasmodium: falciparum, ovale, vivax and malariae. The disease is transmitted to humans via the female Anopheles mosquito. Plasmodium sporozoites from Anopheles saliva invade and multiply in hepatic cells. Lysis of the hepatic cells leads to release of the parasite into the blood stream. The parasite subsequently invades red blood cells (RBCs) and feeds on hemoglobin. RBC lysis follows and is accompanied by fever. In addition to fever, patients often present with headache, nausea, chills, fatigue, abdominal pain and anemia. Unlike the other strains, P. falciparum can cause organ dysfunction and death. Although the history and physical with particular focus on travel history to endemic malaria regions suggests the diagnosis, the gold standard test is microscopic evaluation of a thick and thin smear of blood. Peripheral blood is stained with Giemsa or Wright stains that reveal the presence of the parasite. A complete blood count (A) will likely show anemia in patients with malaria but this is a non-specific finding. ELISA testing (B) is used for the diagnosis of a number of infectious pathogens but not for malaria. VDRL (D) is used in the diagnosis of syphilis.

A 16-year-old girl presents complaining of pain behind her left ear. She thought the pain was due to an ear infection and took three of her boyfriend's leftover antibiotic tablets without seeing her primary care provider. Her ear pain improved for a couple of days, but now she is complaining of fever and discharge from the external auditory canal. Her vitals are T 38.4°C, BP 120/80, HR 108, and RR 18. On physical examination, she has postauricular tenderness, swelling, and erythema. You note purulent otorrhea through a perforated tympanic membrane. Which of the following is the most appropriate first step in management? Administer topical antibiotic suspension containing ciprofloxacin, and place an ear wick Consult an ENT physician CT scan and intravenous antibiotics Restart oral antibiotic therapy and treat for ten days Saline irrigation and suctioning of the external auditory cana

Correct Answer ( C ) Explanation: This patient has acute mastoiditis, an infection of the mastoid air cells that usually results from extension of untreated or inadequately (as in this case) treated otitis media. Patients typically present with fever, chills, postauricular (mastoid) erythema and tenderness, and discharge from the external auditory canal. It is critical to identify and treat mastoiditis because untreated mastoiditis can progress to meningitis, encephalitis, sinus thrombosis, brain abscess, facial nerve palsy, and sepsis. A contrast-enhanced CT scan of the head with inclusion of the petrous portion of the temporal bones should be obtained to confirm the diagnosis (primarily through visualization of fluid in the mastoid air cells) and identify the presence of any complications. Patients with acute mastoiditis should be started on intravenous antibiotics, and most require admission. However, chronic mastoiditis can be treated on an outpatient basis with oral antibiotics and ENT follow-up. Placement of an ear wick and topical antibiotic therapy (A) with a fluoroquinolone is appropriate treatment for otitis externa, not mastoiditis. An ENT physician (B) should be consulted, but this is not the first step in the management process. Patients should first have the diagnosis confirmed by CT scan followed by treatment with intravenous antibiotics. Oral antibiotics (D) are adequate for the treatment of chronic, not acute, mastoiditis. Saline irrigation (E) and suctioning is used in patients with otitis externa to allow for a better evaluation of the external auditory canal.

A 58-year-old man with diabetes mellitus and hypertension presents with a 6-month history of generalized pruritus. He reports that he scratches frequently. On examination his skin is dry and scaly. He has multiple linear excoriations and thickened skin on his forearms, legs, and neck. Which one of the following is the most likely cause of his pruritus? Chronic urticaria Contact dermatitis Lichen simplex chronicus Scabies

Correct Answer ( C ) Explanation: This patient has lichen simplex chronicus, consisting of lichenified plaques and excoriations that result from excessive scratching. On physical exam, one or more slightly erythematous, scaly, well-demarcated, lichenified, firm, rough plaques with exaggerated skin lines are noted. Atopic dermatitis results in a higher probability of developing lichen simplex chronicus. Psychological factors appear to play a role in the development or exacerbation of lichen simplex chronicus. Anxiety has been reported to be more prevalent in patients with lichen simplex chronicus. Insect bites, scars, postherpetic zoster, xerosis, venous insufficiency and asteatotic eczema are common factors. An elevated serum immunoglobulin E level occasionally supports the diagnosis of an underlying atopic diathesis. Perform potassium hydroxide examination and fungal cultures to exclude tinea cruris or candidiasis in patients with genital lichen simplex chronicus. Treatment is aimed at reducing pruritus and minimizing existing lesions because rubbing and scratching cause lichen simplex chronicus. Location, lesion morphology, and extent of the lesions influence treatment. For example, a thick psoriasiform plaque of lichen simplex chronicus on a limb is commonly treated with a highly potent topical corticosteroid or intralesional corticosteroids, whereas vulvar lesions are more commonly treated with a mild topical corticosteroid or a topical calcineurin inhibitor. Widespread lesions are more likely to require systemic treatment or total body phototherapy. Contact dermatitis (B) is usually associated with direct skin exposure to an allergen or irritant and is typically localized to the area of exposure. Chronic urticaria (A) causes a typical circumscribed, raised, erythematous lesion with central pallor. Scabies (D) lesions are small, erythematous papules that are frequently excoriated.

A 14-year-old boy presents with right eye redness for one day. Examination reveals conjunctival injection, yellow discharge, and crusting. His visual acuity is normal, intraocular pressure is 12 mm Hg, and there is no fluorescein uptake. What management is indicated? Acetazolamide Homatropine drops Polymyxin B plus trimethoprim drops Timolol drops

Correct Answer ( C ) Explanation: This patient presents with acute conjunctivitis and should be treated with a topical antibiotic. Conjunctivitis describes the presence of conjunctival inflammation manifested as eye redness and discharge. The majority of cases of conjunctivitis are viral in origin and do not require specific treatment. However, it is often difficult to differentiate between viral and bacterial conjunctivitis. Topical antibiotics are associated with benefits in patients with bacterial conjunctivitis. Patients who use topical antibiotics have higher clinical remission rates. Haemophilus influenzae, Streptococcus pneumoniae, and Staphylococcus aureus are the most common bacterial pathogens. Polymyxin B plus trimethoprim solution is the most commonly used and most extensively tested topical antibiotic. Patients with conjunctivitis should have a normal visual acuity and normal intraocular pressures (10 - 15 mm Hg). Acetazolamide (A) may be useful in the treatment of acute angle closure glaucoma as it can reduce intraocular pressure. Homatropine drops (B) can be used to induce mydriasis. Timolol drops (D) are used in the emergent treatment of acute angle closure glaucoma.

Which of the following statements is correct regarding Osgood-Schlatter disease? Disorder requires surgical intervention Occurs commonly in women over age 30 Pain occurs over the tibial tuberosity Requires MRI for confirmation of diagnosis

Correct Answer ( C ) Explanation: Osgood-Schlatter disease manifests as pain over the tibial tubercle in a growing child. The patellar tendon inserts into the tibia tubercle, which is an extension of the proximal tibial epiphysis. Osgood-Schlatter disease is likely a traction apophysitis of the tibial tubercle growth plate and the adjacent patellar tendon. It occurs during late childhood or adolescence, especially in athletes, and is likely due to repetitive tensile microtrauma. There is often increased prominence of the tibia tubercle that is also firm. Radiographs can be normal or may show small spicules of heterotopic ossification anterior to the tibial tuberosity. This disorder is self-limited in most patients and resolves with skeletal maturity. Rest, restriction of activities and, occasionally, a knee immobilizer may be necessary, combined with an isometric and flexibility exercise program as this is a non-surgical condition (A). Reassurance is important, because some patients and parents fear that the swollen tubercle may be a sign of malignancy. It occurs between the ages of 10 and 15 years old (B); the onset in girls is approximately 2 years sooner than in boys. It is more common in boys. Radiographs (D) are usually the only diagnostic studies necessary and not required for confirmation of diagnosis.

A 17-year-old boy presents with dysuria and urethral discharge. He reports multiple female partners, but none of them are experiencing similar symptoms. Which of the following tests results are most likely to be present? Flagellated, motile protozoans on smear of a urethral swab Gram negative intracellular diplococci on smear of a urethral swab Positive HSV-2 IgG titer Positive urine Chlamydia PCR

Correct Answer ( D ) Explanation: Chlamydia trachomatis is the second most common sexually transmitted infection in both males and females in the United States (HPV is the first). The most common manifestation of chlamydial infection in males is urethritis, or the infection may be asymptomatic. However, infection may also lead to epididymitis, prostatitis, proctitis, and reactive arthritis. Nucleic acid amplification tests (NAAT)'s, such as polymerase chain reactions (PCR's) are sensitive and specific tests to establish a diagnosis of C.trachomatis. In the above case, a positive urine Chlamydia PCR is likely to be present. Flagellated, motile protozoans on a smear of a urethral swab (A) indicates trichomoniasis. Most men with trichomoniasis are asymptomatic, and the infection may persist for months without symptoms, or it may self-resolve. Asymptomatic men are still contagious until the infection is treated or self-resolves. Culture and PCR of urethral swabs are the preferred methods of diagnosis. Wet preps from male urethral specimens are not sufficiently sensitive to rule out trichmoniasis. Gram negative intracellular diplococci on a smear of a urethral swab (B) describes infection with Neisseria gonorrhoeae. While gonorrhea is a common cause of urethritis in males, it is less common than chlamydial infection. Moreover, smears of urethral swabs are insensitive for establishing a diagnosis. Gonorrheal PCR is a preferred method of diagnosis, as it is both sensitive and specific. A positive HSV-2 IgG titer (C) indicates that the patient has been exposed to HSV-2 in the past. However, it does not necessarily indicate current infection. Moreover, the patient's dysuria and urethral discharge are consistent with urethritis, which is an uncommon presentation of genital herpes. Moreover, he lacks the painful vesicles and ulcerations of genital herpes infections.

A ten-year-old boy collapses on the football field and is quickly evaluated by the sports medicine doctor. He had been practicing on the field for approximately two hours, without any rest. His temperature is 105°F and his skin is sweaty. On testing his mental status, he can say his name and age but does not know where he is. Which of the following is the most appropriate initial treatment for this boy? Cool the body with fans Offer oral rehydration solution Provide cold water to drink Whole body cold-water immersion

Correct Answer ( D ) Explanation: Heat stroke is a severe illness with mortality rates of up to 50%. It is characterized by CNS abnormalities and may lead to multi-organ damage. It may occur in the setting of intense physical activity during sports play. Rectal temperature is usually > 104°F, and patients will have profuse sweating. In contrast, "classic" heat stroke is seen in elderly patients and is usually a slower onset, with the physical exam notable for dry, hot skin. Treatment of children with heat stroke consists of immediate whole body cooling, usually performed by cold water immersion. Rapid cooling should be continued until the temperature decrease to 101 or 102°F. Careful monitoring of the airway, breathing, circulation, temperature, and CNS status is crucial. IV fluids should also be initiated as soon as possible, at a rate of 800 ml/m2 in the first hour. Heat exhaustion (A) is a moderate heat illness characterized by a core body temperature of 100-103°F. CNS dysfunction is usually not present, or may be mild. Patients may exhibit headache, gastrointestinal symptoms such as nausea or vomiting, dizziness, weakness, piloerection, orthostasis and possibly syncope. In this case, play should be stopped and the patient should be moved to a cool environment. The body can be cooled with fans, and ice should be placed over the groin or axillae. Oral rehydration (B) should be offered in the setting of heat exhaustion, but if not tolerated, IV fluids should be initiated. If the patient develops signs of heat stroke, he should be transported immediately to an emergency facility. Heat cramps are the most common heat injury and usually occur in the setting of mild dehydration and salt loss. They often affect the calf and hamstring muscles. Treatment consists of oral rehydration with electrolyte solutions and gentle stretching. Cold water (C) should be offered to all children participating in physical activity in the heat. This is appropriate for all children exercising for less than 1 hour. However, fluids with electrolytes and sugar should be offered for those exercising greater than 1 hour, as drinking only water may put them at risk for hyponatremia.

A 17-year-old woman presents to the ED complaining of sore throat and cough for two days. Her review of symptoms is positive for fever, headache, and dysphagia. Vital signs are T 38.5°C, BP 110/80 mm Hg, and HR 115 bpm. Physical exam reveals bilateral tonsillar enlargement with exudates, midline uvula, and tender anterior cervical lymphadenopathy. Which of the following is the most appropriate next step in management? Aspirate the peritonsillar area using an 18-gauge needle and treat empirically with antibiotics No diagnostic testing; treat empirically with antibiotics Obtain a throat culture and have the patient follow-up in 48 hours for the results Perform a rapid antigen testing for Group A ß-hemolytic Streptococcus (GABHS) and treat with antibiotics if positive

Correct Answer ( D ) Explanation: Most cases of acute pharyngitis have a viral etiology. However, 15% of pharyngitis is caused by GABHS. The modified Centor criteria are used to predict the likelihood of streptococcal pharyngitis and assist in the decision to empirically treat or not treat with antibiotics. The patient has tonsillar exudates (+1), tender cervical lymphadenopathy (+1), and fever (+1), for a score of 3. Following the algorithm, a rapid antigen test should be obtained and, if positive, empiric treatment with antibiotics is indicated. Peritonsillar aspiration (A) or incision and drainage is performed in patients with suspected peritonsillar abscess. This should be considered in an ill-appearing patient with fever, sore throat, trismus, and displacement of the uvula toward the unaffected tonsil. A rapid antigen test should be performed first (B and C) because if it is positive, empiric treatment should be initiated. However, if it is negative, a culture is recommended, not treatment.

Which of the following is the most common dysrhythmia associated with the diagnosis of pulmonary embolism? Atrial fibrillation AV-nodal reentrant tachycardia Multifocal atrial tachycardia Sinus tachycardia

Correct Answer ( D ) Explanation: The 12-lead ECG is best used in the setting of suspected PE to provide information about alternative diagnoses such as pericarditis or cardiac ischemia. This is because normal or near normal ECGs with sinus tachycardia or nonspecific ST-T wave changes may be seen up to 30% of the time. When PE causes ECG changes, it is usually the result of acute or subacute pulmonary hypertension. The most common dysrhythmia seen in this setting is sinus tachycardia. Other findings of right-heart strain include T wave inversion in the anterior leads (V1-V4), the S1Q3T3 pattern, and incomplete or complete right bundle branch block. Atrial fibrillation (A) can be seen with pulmonary embolism but is less common than sinus tachycardia. Atrioventricular nodal reentry tachycardia (AVNRT) (B) is the most common type of reentrant supraventricular tachycardia. Pulmonary embolism is an unlikely cause of AVNRT Multifocal atrial tachycardia (MAT) (C) is a dysrhythmia that is seen in a variety of clinical disorders. It is more common in the elderly and is associated with COPD and hypoxia. In addition to a heart rate greater than 100 beats per minute, the characteristic ECG feature is variability in P wave morphology, with each unique P wave morphology felt to indicate a different site of atrial origin. Patients with multiple P wave morphologies but a normal heart rate are considered to have a

A 62-year-old previously healthy, nonsmoking man presents to your office with questions about his cholesterol. His recent lab results include total cholesterol 245 mg/dL, high- density lipoprotein 38 mg/dL, low-density lipoprotein 165 mg/dL and triglycerides 250 mg/dL. Which of the following is the most appropriate initial therapy? Fenofibrate Fish oil Gemfibrozil Lovastatin

Correct Answer ( D ) Explanation: The American College of Cardiology and the American Heart Association (ACC/AHA) released a new set of guidelines in November 2013 for treatment of hyperlipidemia with the focus being to reduce the risk of atherosclerotic cardiovascular disease (ASCVD) in adults. Cardiovascular risk is estimated based on age, total cholesterol, high-density lipoprotein (HDL) cholesterol, low-density lipoprotein (LDL) cholesterol, gender, smoking status, systolic blood pressure and the presence or absence of diabetes. Several online calculators are available to determine risk. Four groups have been identified as benefiting from statin therapy: patients with ASCVD, patients with LDL levels greater than or equal to 190 mg/dL, patients aged 40-75 years with diabetes and an LDL level of 70-189 mg/dL, and patients with an LDL level of 70-189 mg/dL and a 10-year ASCVD risk of greater than or equal to 7.5%. The starting point to reduce the risk of ASCVD is lifestyle modifications such as regular physical exercise, heart-healthy diet, smoking cessation and maintaining a healthy weight. First-line treatment for patients meeting criteria for treatment of hypercholesterolemia is statin therapy, including lovastatin. This patient meets criteria for treatment based on his ASCVD score of > 7.5%. Fenofibrate (A) and gemfibrozil (C) are used in the treatment of hypertriglyceridemia. Fish oil (B) is recommended for patients with refractory hypertriglyceridemia and not as first line for hypercholesterolemia.

A 42-year-old woman complains of two days of pain and swelling in the right submandibular area. She complains of dry mouth and worsening of the swelling and pain during mealtime. Which of the following is the first-line treatment for this condition? Antihistamines Dilation and incision Oral antibiotics Sialogogues

Correct Answer ( D ) Explanation: This patient has obstructive sialoadenitis, which occurs from outflow obstruction by a stone or calculus in the salivary gland or duct. The submandibular location is most commonly involved because it is has more viscous secretions and runs an uphill course. Patients with sialolithiasis note xerostomia (dry mouth) along with increasing swelling and pain during mealtime. Most salivary stones pass spontaneously. To aid in passage, patients should be started on sialogogues (e.g., sour lozenges), which stimulate salivary secretions and help expel the stone. Palpable stones may also be "milked" from the duct, if they are distal enough, by gentle stroking in a posterior to anterior direction. Antihistamines (A) can worsen this condition by decreasing saliva production and are contraindicated. If the sialoadenitis does not resolve with conservative therapy, dilation and incision (B) of the salivary duct is required to remove the stone. Oral antibiotics (C) are not required in simple obstructive sialoadenitis. However, when suppurative sialoadenitis is present, oral antibiotics with staphylococcal coverage are recommended.

63-year-old man with chronic alcoholism presents with headache, fever and neck pain. Physical examination reveals neck stiffness. What antibiotics should be administered? Ampicillin alone Ceftriaxone alone Ceftriaxone and vancomycin Ceftriaxone, vancomycin and ampicillin

Correct Answer ( D ) Explanation: This patient presents with signs and symptoms concerning for meningitis and should receive antibiotics covering the most likely organisms including Listeria monocytogenes. Meningitis describes an inflammation of the meninges. Patients typically present with headache, fever, neck pain and possibly, altered mental status. Diagnosis is made by analysis of cerebrospinal fluid (CSF). The most useful diagnostic tests are a CSF cell count and gram stain. Gram stain may reveal the causative organism. However, antibiotics should not be withheld while either waiting to perform the lumbar puncture or waiting for the CSF results. Patients with a high suspicion for meningitis should have appropriate antibiotics started as soon as possible. The most common organisms in healthy adults are Streptococcus pneumoniae and Nisseria meningitides. Both of these organisms are covered by ceftriaxone however there is a small percentage of resistant S. pneumoniae requiring treatment with vancomycin. Other organisms are more common based on age group and comorbid conditions. Listeria monocytogenes is a more common cause of meningitis in very young infants, elderly patients and those with chronic alcoholism or immunosuppression. L. monocytogenes is typically treated with ampicillin. Although L. monocytogenes is more common in this patient population, it is not more common than S. pneumoniae and N. meningitides. Therefore, treatment with ampicillin alone (A) is not indicated. Ceftriaxone (B) is adequate for most strains of S. pneumoniae and N. meningitides but does not cover resistant S. pneumoniae. Neither ceftriaxone or vancomycin (C) treats L. monocytogenes.

A patient presents to your primary care clinic for a new patient evaluation. Which of the following physical examination findings would prompt you to order a lipid panel? Cholesteatoma Corneal ulcer Pitting edema Xanthoma

orrect Answer ( D ) Explanation: The primary dyslipidemias include familial types of combined hyperlipidemia, hypercholesterolemia, defective apoprotein-B100, hypertriglyceridemia and dysbetalipoproteinemia, which can be differentiated by which lipoprotein abnormalities predominate (namely total cholesterol, HDL, LDL and triglycerides). Secondary dyslipidemias occur with lifestyle choices (obesity, alcohol, tobacco, inactivity), endocrine disease (DM2, thyroid disorders, Cushing's syndrome), renal disease (nephrotic syndrome, renal failure), hepatic disease (hepatitis, liver failure, cholestasis) and medications (thiazides, estrogen, protease inhibitors and beta-blockers). If history reveals any of the above conditions, one should investigate for the presence of dyslipidemia, especially if no baseline labs are available. In addition, there are some physical exam findings which support the diagnosis of dyslipidemia. Achilles, elbow and hand tendon xanthomas are common when LDL >300 mg/dL. Eruptive xanthomas, acne-like lesions which occur on extensor surfaces or body-wide, imply hypertriglyceridemia > 1000mg/dL. Xanthelasma and corneal arcus may also be present in those with dyslipidemia. Cholesteatoma (A) is a middle ear or mastoid process destructive growth comprised of keratinized squamous epithelium. It has no correlation with the cholesterol disorders. Corneal ulcers (ulcerative keratitis) (B) represents traumatic, chemical, contact lens or infectious destruction of the corneal epithelium. Compare this to a corneal arcus, which is a greyish-white, peripheral corneal ring opacity commonly seen in the elderly, but if seen in young patients, it implies underlying dyslipidemia. Pitting edema (C) suggests cardiac, hepatic or renal issues. This finding supports ordering a basic metabolic or liver panel, not a lipid panel.


Related study sets

Chapter 1 -Intro Fundamentals of Law for Health Informatics & Information Management

View Set

Boca Heat Transfer CH 8-11(Internal Flow, Free Convection, Boil/cond, Heat Exchangers)

View Set

Healthy Wealthy and Wise Chapter 5/5a Quiz

View Set

cells unit 2: cell division and genetics

View Set